UPSC Mock Paper-1 Mock-1

You might also like

Download as pdf or txt
Download as pdf or txt
You are on page 1of 28

Mock Test Paper-01

(UPSC Civil Services (Pre) Examination)


General Studies Paper-I
Time : 2 Hours Maximum Marks: 200

GENERAL INSTRUCTIONS
1. This Test Booklet contains 100 items (Questions). Each item comprises four responses (answers). In case you
feel that there is more than one correct response, mark the response which you consider the best. In any
case, choose ONLY ONE response for each item.
2. You have to mark all your responses ONLY on the separate OMR Sheet provided.
3. All items carry equal marks.
4. Attempt all items.
5. Your total marks will depend only on the number of correct responses marked by you in the OMR sheet.
6. For every incorrect response 1/3rd of the allotted marks will be deducted.
7. If a question is left blank i.e., no answer is given by the candidate, there will be no penalty for that question.

1. Consider the following statements: 2. In Golakanath Case (1967), the Supreme court
1. In Berubari v/s Union of India (1960) Case, the had pronounced that the Parliament cannot
Supreme Court states that the Preamble is not amend the Fundamental Rights to give effect to
part of the Constitution. the Directive Principles of State Policy.
2. In Kesavananda Bharati (1973) Case, the Which of the statements given above is/are correct?
Supreme Court states that the Preamble is a part (a) 1 only (b) 2 only
of the constitution. (c) Both 1 and 2 (d) Neither 1 nor 2
3. In LIC of India (1995) Case, the Supreme Court 4. Consider the following statements regarding the
states that the Preamble is an integral part of the Central Vigilance Commission (CVC).
Constitution. 1. It was a statutory body established in 1964.
Which of the statements given above are correct? 2. It was established on the recommendations of
(a) 1 and 2 only (b) 2 and 3 only the Sarkaria Commission.
(c) 1 and 3 only (d) 1, 2 and 3 3. Its jurisdiction extends to members of the All
2. Which of the following provisions are Fundamental India Services serving in connection with the
Duties under Part IVA of the Indian Constitution? affairs of the Union.
1. To abide by the constitution and respect its ideals Which of the statements given above is/are correct?
and institutions. (a) 1 and 2 only (b) 1 and 3 only
2. To protect the integrity, sovereignty and unity of (c) 3 only (d) 1, 2 and 3
India. 5. Consider the following statements:
3. To cherish and preserve the rich national heritage 1. The 25th Constitutional Amendment Act
of our composite culture. facilitated the appointment of the same person
4. To promote the spirit of common brotherhood as a governor for two or more states.
amongst all the people of India. 2. The 91st Amendment Act of 2003 has made some
Select the correct answer using the code given below: provisions to limit the size of the Council of
(a) 1 and 2 only (b) 3 and 4 only Ministers.
(c) 1, 2 and 3 only (d) 1, 2, 3 and 4 only Which of the statements given above is/are correct?
3. Consider the following statements regarding (a) 1 only (b) 2 only
Fundamental Rights and Directive Principles of (c) Both 1 and 2 (d) Neither 1 nor 2
State Policy (DPSP). 6. The 86th Constitutional Amendment Act of 2002
1. Article 39 (b) and Article 39 (c) are two Directive made amendments to which one of the following?
Principles of State Policies (DPSPs) have been 1. Directive Principles of State Policy
given precedence over Fundamental Rights 2. Fundamental Rights
Article 14 and Article 19. 3. Fundamental Duties
2 UPSC CSE 15 MOCK TEST PAPERS GENERAL STUDIES PAPER I

Select the correct answer using the code given below: 12. Consider the following statements regarding
(a) 1 only (b) 1 and 2 only President Rule in a state:
(c) 1 and 3 only (d) 1, 2 and 3 1. Article 365 empowers the President to issue a
7. Consider the following statements regarding proclamation if he is satisfied that a situation has
National Emergency with respect to fundamental arisen in which the government of a state cannot
rights: be carried on in accordance with the provisions
1. Article 19 is automatically suspended when a of the Constitution.
National Emergency is proclaimed. 2. Article 356 says that whenever a state fails to
2. Article 358 authorizes the president to suspend comply with or give effect to any direction from
the right to move any court for the enforcement the Centre, it will be lawful for the president
of Fundamental Rights during a National to hold that a situation has arisen in which
Emergency. the government of the state cannot be carried
Which of the statements given above is/are correct? on in accordance with the provisions of the
(a) 1 only (b) 2 only Constitution.
(c) Both 1 and 2 (d) Neither 1 nor 2 Which of the statements given above is/are correct?
8. Under the Indian Constitution, concentration of (a) 1 only (b) 2 only
wealth violates (c) Both 1 and 2 (d) Neither 1 nor 2
1. The Right to Equality 13. With reference to the District Development Councils
2. The Directive Principles of State Policy (DDCs), consider the following statements:
3. Fundamental duties 1. The electoral process of DDC allows for
Select the correct answer using the code given below: reservations for Scheduled Castes and Scheduled
(a) 1 only (b) 2 only Tribes only.
(c) 1 and 3 only (d) 1, 2 and 3 2. The members of DDC are elected from each
9. Consider the following statements regarding the district.
National Commission for Minorities: Which of the statements given above is/are correct?
1. It is a seven-members body, and all of them shall (a) 1 only (b) 2 only
be among the minority communities. (c) Both 1 and 2 (d) Neither 1 nor 2
2. Each member holds office for a period of five 14. Consider the following statements with respect to
years. the expenditure limit for a candidate in contesting
3. Jains were also notified as a minority community
elections.
since 1992.
1. The Election Commission of India imposes limits
Which of the statements given above is/are correct?
on the expenditure incurred by a candidate
(a) 1 only (b) 2 only
as well as political parties on their election
(c) 1 and 3 only (d) 1, 2 and 3
campaign.
10. Consider the following statements regarding the 2. The expenditure limits range from ` 20 lakh to
Public Accounts Committee: ` 28 lakh for assembly elections, and from ` 54
1. All the members of the Public Accounts lakh to ` 70 lakh for Lok Sabha elections.
Committee are from the Lok Sabha only. Which of the statements given above is/are correct?
2. The Chairman is appointed by the Speaker of (a) 1 only (b) 2 only
Lok Sabha. (c) Both 1 and 2 (d) Neither 1 nor 2
3. It’s chief function is to examine the audit report
of the Comptroller and Auditor General (CAG). 15. Which of the following is/are the features of
Which of the statements given above is/are correct? Fundamental Rights guaranteed by the Indian
(a) 1 only (b) 2 and 3 only Constitution?
(c) 1 and 3 only (d) 1, 2 and 3 1. They are not absolute and sacrosanct.
2. They are available against the arbitrary action of
11. Consider the following statements regarding the
a state only.
Attorney General of India:
3. They are enforceable only in the High Courts
1. Article 76 deals with for the Attorney General of
and the Supreme Court.
India.
Select the correct answer using the code given below:
2. Tenure of Attorney General of India is fixed and
(a) 3 only (b) 1 and 3 only
mentioned in the constitution.
(c) 1 and 2 only (d) 1, 2 and 3
3. Enjoys privileges and immunities equivalent to a
member of Parliament. 16. Which of the following requires the special majority
Which of the statements given above is/are correct? under the Constitutional amendment (Article 368)
(a) 1 only (b) 2 and 3 only of the constitution?
(c) 1 and 3 only (d) 1, 2 and 3 1. Election of the President and its manner
MOCK TEST PAPER - 01 3

2. Delimitation of constituencies Which of the above statements are correct?


3. Changes in representation of states in Parliament (a) 1 only (b) 2 only
Select the correct answer using the code given below: (c) Both 1 and 2 (d) Neither 1 nor 2
(a) 1 and 2 only (b) 2 and 3 only
22. Consider the following statements regarding AT-1
(c) 1 and 3 only (d) 1, 2 and 3
Bonds:
17. Consider the following statements with respect to 1. AT-1 bonds are a type of unsecured perpetual
the Finance Commission: bonds that banks issue to shore up their core
1. It is a constitutional body formed under Article capital base to meet the Basel-III norms.
324 of the constitution. 2. These bonds offer higher returns to investor, but
2. It’s members are eligible for reappointment. carry a higher risk as well.
3. Constitution authorizes the Parliament to
Which of the above statements is/are correct?
determine the qualifications of members of the
(a) 1 only (b) 2 only
finance commission.
(c) Both 1 and 2 (d) Neither 1 nor 2
Which of the statements given above is/are correct?
(a) 1 only (b) 2 and 3 only 23. Consider the following statements regarding
(c) 1 and 2 only (d) 1, 2 and 3 Payment banks:
1. Payments banks can provide ATM and debit
18. Consider the following statements regarding the
cards but cannot provide online and mobile
GST council:
1. The GST Council is a constitutional body banking.
under Article 279A and was introduced by the 2. Payments banks are unable to provide loans or
Constitutional amendment act 103rd 2016. credit cards.
2. The Finance secretary is the chairman of the council. 3. Payment Banks can accept deposit upto
Which of the above statements are correct? ` 5,00,000 per person.
(a) 1 only (b) 2 only Which of the statements given above is/are correct?
(c) Both 1 and 2 (d) Neither 1 nor 2 (a) 1 only (b) 2 only
(c) 1 and 2 only (d) 1, 2 and 3
19. Consider the following statements regarding PM-
KISAN: 24. Consider the following statements regarding the
1. Under the scheme, the Centre transfers an Indian Renewable Energy Development Agency:
amount of ` 6000 per year in three equal 1. It is a miniratna company under the Ministry of
instalments directly into the bank accounts of all New and Renewable Energy.
landholding farmers having less than 2 hectares 2. It was set up as a specialized non-banking
of land. finance agency for the renewable energy sector.
2. It is implemented by the Ministry of Agriculture Which of the above statements is/are correct?
and Farmers Welfare. (a) 1 only (b) 2 only
3. It is a Central Sector Scheme with 100% funding (c) Both 1 and 2 (d) Neither 1 nor 2
from the Government of India.
25. Consider the following statements regarding the
Which of the statements given above is/are correct?
(a) 1 only (b) 2 and 3 only Foreign Contribution Regulation:
(c) 1 and 2 only (d) 1, 2 and 3 1. The Act prohibits receipt of foreign funds by
candidates for elections journalists or newspaper
20. Consider the following statements regarding the
and media broadcast companies.
Food Safety and Standards Authority of India:
2. FCRA is implemented by the Ministry of Finance.
1. It is a statutory body established under the Food
3. It was amended in the year 2010 and 2020.
Safety and Standards Act 2006.
Which of the statements given above is/are correct?
2. The administrative ministry for FSSAI is the
(a) 1 only (b) 1 and 2 only
Ministry of Agreement.
Which of the above statements is/are correct? (c) 1 and 3 only (d) 1, 2 and 3
(a) 1 only (b) 2 only 26. Consider the following statements with reference
(c) Both 1 and 2 (d) Neither 1 nor 2 to the National Bank for Agriculture and Rural
21. Which of the following statements are correct Development (NABARD):
regarding the Index of Industrial Production: 1. It is a statutory body established under the
1. It is used by government agencies including the National Bank for Agriculture and Rural
Ministry of Finance and the Reserve Bank of Development Act 1981.
India for policy-making purposes. 2. It supervises Cooperative Banks and Regional
2. It is compiled and published monthly by the Rural Banks (RRBs).
Central Statistical Organisation (CSO) Ministry 3. It is the apex banking institution to provide
of Statistics and Programme. finance for Agriculture and rural development.
4 UPSC CSE 15 MOCK TEST PAPERS GENERAL STUDIES PAPER I

Which of the statements given above is/are correct? 2. It was adopted under the aegis of the United Nations
(a) 1 only (b) 1 and 2 only Framework Convention on Climate Change.
(c) 1 and 3 only (d) 1, 2 and 3 3. It is not binding on the parties.
27. Consider the following statements regarding Which of the statements given above is/are correct?
African Swine Fever: (a) 1 only (b) 1 and 3 only
1. It is a highly contagious and fatal animal disease. (c) 2 only (d) 1, 2 and 3
2. It is not a threat to human beings since it only 34. Consider the following statements:
spreads from animals to other animals. 1. In Mutualism, both the species get benefited.
Which of the above statements are correct? 2. In Parasitism, only one species benefits and the
(a) 1 only (b) 2 only other is harmed.
(c) Both 1 and 2 (d) Neither 1 nor 2 3. In Amensalism, one species benefits and the
28. Neem tree has acquired industrial importance as a other is unaffected.
source of Which of the above is/are correct?
(a) Biopesticide and anti-fertility compound. (a) 1 only (b) 1 and 2 only
(b) Anti-fertility compound, biofertilizer and anti- (c) 3 only (d) 1, 2 and 3
cancer drug.
35. Consider the following statement regarding Dhole:
(c) Biofertilizer, biopesticide and anti-fertility com-
1. Dhole is a canid native to Central South Asia
pound.
only.
(d) Anti-cancer drug, biopesticide and biofertilizer.
2. IUCN status is critically endangered.
29. Github recently seen in news is 3. Depletion of prey base is a threat to the species.
(a) Open-source software. Which of the above is/are correct?
(b) Military program of the USA. (a) 1 and 2 only (b) 2 only
(c) Covid-19 vaccine. (c) 3 only (d) 1, 2 and 3 only
(d) Supercomputer of China.
36. Consider the following statements regarding corals:
30. Consider the following statements regarding the
1. They are mainly found in tropical oceans and
NISAR mission:
seas.
1. NISAR is a joint Earth-observing mission
2. Most fishes lay eggs in coral colonies.
between NASA and the ISRO.
3. Coral reefs have greater biodiversity than
2. ISRO will provide the spacecraft bus, the second
tropical rainforests.
type of radar (called the S-band radar) the launch
Which of the statements given above is/are correct?
vehicle and associated launch services.
(a) 1 and 2 only (b) 1, 2 and 3
Which of the above statements are correct?
(c) 2 and 3 only (d) 2 only
(a) 1 only (b) 2 only
(c) Both 1 and 2 (d) Neither 1 nor 2 37. Which of the following is true for Source-sink
31. With respect to HIV, consider the following dynamics?
statements: (a) The Indian government has started looking at
1. ELISA test is used to test for HIV antibodies. means to revive the traditional systems of water
2. HIV is not transmitted from person to person. harvesting in the country.
3. It is necessary that a person infected with HIV (b) The computer simulation of exponential
will definitely develop AIDS. economic and population growth with finite
Which of the given above statements is/are correct? resource supplies.
(a) 1 only (b) 1 and 2 only (c) When we conserve and protect the whole
(c) 2 and 3 only (d) 1, 2 and 3 ecosystem, its biodiversity at all levels is protected.
32. Why is a plant called asafoetida often mentioned in (d) A theoretical model used by ecologists to describe
the news? how variation in habitat quality may affect the
(a) Its extract is widely used in cosmetics. population growth or decline of organisms.
(b) It tends to reduce the biodiversity in the area in 38. With reference to the International Solar Alliance
which it grows. (ISA), consider the following statement:
(c) Its extract is used in the synthesis of pesticides. 1. It was jointly launched by India and South
(d) It has medicinal properties. Africa.
33. With reference to the Convention on International 2. Its headquarter is in Gurugram, India.
Trade in Endangered Species of Wild Fauna and 3. After the United Nations, it is the largest
Flora (CITES), consider the following statements: grouping of states world-wide.
1. It aims to ensure that international trade in Which of the above is/are correct?
specimens of wild animals and plants do not (a) 1 only (b) 1 and 2 only
threaten their survival. (c) 2 and 3 only (d) 1, 2 and 3 only
MOCK TEST PAPER - 01 5

39. BEEG recently seen in news is: 2. Iron tools were used to plough fields and as
(a) Seed balls sickles to harvest the crops.
(b) a variety of rice 3. Indus people were the earliest people to produce
(c) pesticide cotton.
(d) Genetically modified grapes 4. Ragi were grown extensively in the Northwest
40. Recently, the Union Environment minister launched region.
Surakshya portal on: Select the correct answer using the code given
below:
(a) Forest conservation.
(a) 1, 2 and 3 only (b) 1 and 3 only
(b) Conservation of tigers.
(c) 2 and 4 only (d) 1, 3 and 4 only
(c) Human elephant conflict.
(d) e-health services during covid-19. 46. Magadha emerged as the most powerful of the
sixteen mahajanapadas. Which of the following
41. Recently, the paper titled ‘‘SPOTTED” in ‘Illegal
were the prominent reason for their success?
Wildlife Trade: A Peek into Ongoing Poaching and
1. Elephants were an important part of the
Illegal Trade of Leopards in India’ was released by
Magadhan army.
(a) CITES (b) WWF
2. Presence of iron mines which provided resources
(c) TRAFFIC India (d) PETA
for tools and weapons.
42. Consider the following pairs: 3. The Ganga and its tributaries provided a means
Founder King Dynasty of cheap and convenient communication.
Select the correct answer using the code given below:
1. Pushyamitra Founder of Sunga dynasty (a) 1 and 3 only (b) 2 only
sunga
(c) 1 and 2 only (d) 1, 2 and 3
2. Devabhuti Founder of Kanva dynasty
47. Consider the following statements regarding
3. Simuka Founder of Satavahana ancient rituals:
dynasty
1. In Rajasuya Sacrifice Chariot race, a royal chariot
4. Chandragupta Founder of Gupta dynasty was made to win the race against his kinsmen.
Select the correct code. 2. Ashvamedha Sacrifice deals with unquestioned
(a) Only one of them is correct. control over the area on which the royal horse
(b) Only two of them are correct. ran uninterrupted.
Which of the statements given above is/are correct?
(c) Only three of them are correct.
(a) 1 only (b) 2 only
(d) All four are correct.
(c) Both 1 and 2 (d) Neither 1 nor 2
43. Consider the following pairs: 48. Consider the following statements regarding
Ruler Time period Mohiniyattam Dance:
1. Rajiya 1136 - 1140 1. It is one of the classical dances from the state of
2. Iltutmish 1211 - 1236 Kerala.
3. Akbar 1607 - 1647 2. This dance form is related to lord Shiva.
Which of the statements given above is/are correct?
4. Ibrahim Lodi 1517 - 1526
(a) 1 only (b) 2 only
Select the correct code. (c) Both 1 and 2 (d) Neither 1 nor 2
(a) Only one of them is correct. 49. Consider the following statements:
(b) Only two of them are correct. 1. Jataka stories deals with the previous births of
(c) Only three of them are correct. Buddha in the form of human only.
(d) All four are correct. 2. Jataka stories were depicted on the railings and
44. Consider the following pairs: torans of the stupas.
Which of the statements given above is/are correct?
Names Related Personalities (a) 1 only (b) 2 only
1. Maitreya Future Buddha (c) Both 1 and 2 (d) Neither 1 nor 2
2. Chakrapani Samanta
50. With reference to Kathak Dance, consider the
3. Amitabha Sukhavati following statements:
4. Avalokteshwara Padmapani 1. It is one of the eight major forms of Indian
Select the correct code. classical dance.
(a) Only one of them is correct. 2. It is the only classical dance of India having links
(b) Only two of them are correct. with Muslim culture.
(c) Only three of them are correct. Which of the statements given above is/are correct?
(d) All four are correct. (a) 1 only (b) 2 only
(c) Both 1 and 2 (d) Neither 1 nor 2
45. Consider the following statements regarding the
Indus valley civilization: 51. Consider the following statements:
1. The Indus people harvested wheat, barley, rai, 1. The first European power to occupy Pondicherry
peas, rice and mustard. were the French.
6 UPSC CSE 15 MOCK TEST PAPERS GENERAL STUDIES PAPER I

2. Fort William was the first fort constructed by the 3. The Simon Commission report recommended
British in India. dominion status for British India.
Which of the statements given above is/are correct? Which of the statements given above are correct?
(a) 1 only (b) 2 only (a) 1 and 2 only (b) 2 only
(c) Both 1 and 2 (d) Neither 1 nor 2 (c) 1 and 3 only (d) 1, 2 and 3
52. Which of the statements given above is/are correct? 58. Consider the following statements regarding the
1. Dutch were the last to come to pre-independence Cabinet Mission Plan:
India as traders. 1. The Cabinet Mission recommended an
2. Portuguese were the first to come to pre- undivided India.
independence India as traders. 2. It provided that the central government in Delhi
(a) 1 only (b) 2 only would have powers over defence, foreign affairs
(c) Both 1 and 2 (d) Neither 1 nor 2 and communications.
53. Arrange the following in the correct order. Which of the statements given above is/are correct?
1. The 3rd battle of Panipat (a) 1 only (b) 2 only
2. The Battle of Plassey (c) Both 1 and 2 (d) Neither 1 nor 2
3. The Battle of Buxar
59. Arrange the following socio-religious reforms
4. The Battle of Madras
chronologically:
(a) 4-3-1-2 (b) 2-3-4-1
1. Foundation of Brahmo Sabha
(c) 3-4-2-1 (d) 4-2-1-3
2. Foundation of Vedanta college
54. Consider the following statements: 3. Establishment of Arya samaj
1. In the Third Battle of Panipat, Ahmad Shah
Select the correct answer using the code given below:
Abdali defeated Ibrahim Lodi.
(a) 3-1-2 (b) 3-2-1
2. In the Third Anglo-Mysore War (1790-92), Tipu
(c) 2-1-3 (d) 1-2-3
Sultan formed an ally with France and invaded
the nearby state of Travancore. 60. Consider the following statements regarding the
3. Mir Jafar entered in a conspiracy with the English Umngot river:
for the defeat of Nawab Siraj-ud-Daulah in the 1. It is a river in Meghalaya.
Battle of Plassey. 2. It is considered to be India’s cleanest river.
Which of the statements given above is/are correct? 3. Dawki Bridge is a suspension bridge over the
(a) 1, 2 and 3 (b) 3 only Umngot River.
(c) 2 and 3 only (d) 1 and 3 only Which of the statements given above are correct?
55. Consider the following statements: (a) 1 and 2 only (b) 2 only
1. Tipu Sultan established embassies with many (c) 1 and 3 only (d) 1, 2 and 3
foreign countries. 61. Which of the following statements with reference to
2. Awadh was removed from power by the British Tanzania is/are correct?
on the pretext of mis-governance. 1. Tanzania is a country in west Africa.
Which of the statements given above is/are correct? 2. Mount Kilimanjaro, Africa’s highest mountain, is
(a) 1 only (b) 2 only located in Tanzania.
(c) Both 1 and 2 (d) Neither 1 nor 2 3. It borders Uganda to the north; Comoro Islands
56. Which of the following are the provisions of the and the Indian Ocean to the east; Mozambique
Gandhi-Irwin Pact of 1931? and Malawi to the south; Rwanda Burundi and
1. The immediate release of all political prisoners the Democratic Republic of the Congo to the
not convicted for violence. west.
2. Government also conceded the right to make salt Which of the statements given above is/are correct?
for consumption. (a) 1 and 2 only (b) 2 only
3. A public inquiry into police excesses was (c) 2 and 3 only (d) 1, 2 and 3
accepted.
Select the correct answer using the code given below: 62. Kowloon peninsula recently seen in news is a
(a) 1 and 3 only (b) 2 only part of:
(c) 1 and 2 only (d) 1, 2 and 3 (a) Madagascar islands
(b) East Ukraine
57. Consider the following statements with reference to
(c) North American Central part
the Simon Commission:
(d) Southern part in the territory of Hong Kong
1. All the members of the Commission were
Englishmen. 63. Consider the following statements regarding the
2. The Commission was appointed to go into the Godavari river:
question of further constitutional reforms in 1. The Godavari is the second largest Peninsular
British India. river system.
MOCK TEST PAPER - 01 7

2. Kumbh Mela also takes place on the banks of the Which of the statements given above is/are correct?
Godavari river in Nashik. (a) 1 only (b) 2 only
3. The Godavari river rises from Trimbakeshwar (c) 1 and 3 only (d) 1, 2 and 3
near Nasik in Maharashtra.
69. Consider the following statements regarding
Which of the statements given above are correct?
Khajuraho Temples:
(a) 1 and 2 only (b) 2 only
1. The temple site is situated within Satpura
(c) 2 and 3 only (d) 1, 2 and 3
Mountain range.
64. Consider the following statements regarding the 2. The monuments include Hindu and Buddhist
New Development Bank: temples.
1. It is a multilateral development bank operated 3. It is built by the Chandela Dynasty.
by the BRICS states. Which of the statements given above is/are correct?
2. In the New Development Bank each participant (a) 1 only (b) 3 only
country will be assigned one vote.
(c) 1 and 3 only (d) 1, 2 and 3
3. In 2018, the NDB received observer status in the
World Bank. 70. With reference to the World Press Freedom Index,
Which of the statements given above are correct? consider the following statements:
(a) 1 and 2 only (b) 2 only 1. India is below Nepal and Sri Lanka but above
(c) 2 and 3 only (d) 1, 2 and 3 Bangladesh in position.
2. It is published by the Reporters Without Borders.
65. Consider the following statements regarding the
Which of the statements given above is/are correct?
North Atlantic Treaty Organization:
(a) 1 only (b) 2 only
1. It is an intergovernmental trade alliance
established by the North Atlantic Treaty (also (c) Both 1 and 2 (d) Neither 1 nor 2
called the Washington Treaty) of 1949 by the 71. Consider the following statements regarding the
United States, Canada and several Western Siachen glacier:
European nations. 1. The Siachen Glacier is located in the western
2. The most recent member state to be added to Karakoram range in the Himalayas.
NATO was North Macedonia. 2. The entire Siachen Glacier with all major passes
Which of the statements given above is/are correct? is currently under the administration of India.
(a) 1 only (b) 2 only Which of the statements given above is/are correct?
(c) Both 1 and 2 (d) Neither 1 nor 2 (a) 1 only (b) 2 only
66. Project DANTAK is related to: (c) Both 1 and 2 (d) Neither 1 nor 2
(a) Conservation of Asian elephant with tusk. 72. Nusantara recently seen in news is
(b) A nutritional scheme for tribal children under 14 (a) An island of Australia where a volcanic eruption
years. took place recently
(c) Providing medical facilities to old citizens who
(b) New capital of Malaysia
are not covered under any social service schemes.
(c) New capital of Myanmar
(d) Infrastructure project between India and Bhutan.
(d) New capital of Indonesia
67. Consider the following statements with reference
73. Consider the following statements regarding the
to President’s Rule under Article 356 of the
Inter-state water dispute:
Constitution:
1. Article 263 provides for the adjudication of inter-
1. There is no maximum period prescribed for
state water disputes.
president rule in the constitution.
2. Parliament may also provide that neither the
2. A proclamation imposing President’s Rule must
be approved by both the Houses of Parliament Supreme Court nor any other court is to exercise
within six months from the date of its issue. jurisdiction in respect of any such dispute.
Which of the statements given above is/are correct? Which of the statements given above is/are correct?
(a) 1 only (b) 2 only (a) 1 only (b) 2 only
(c) Both 1 and 2 (d) Neither 1 nor 2 (c) Both 1 and 2 (d) Neither 1 nor 2

68. Consider the following statements regarding Gold 74. With respect to Sedition Law in India, which of the
Exchange: following statements is/are correct?
1. It is a national platform for buying and selling 1. Sedition is a non-bailable offence.
Electronic Gold Receipts. 2. A person charged under this law is barred from a
2. RBI would regulate the entire ecosystem of the government job.
proposed gold exchange. Which of the statements given above is/are correct?
3. Physical gold deposited at one location can be (a) 1 only (b) 2 only
withdrawn from a different location. (c) Both 1 and 2 (d) Neither 1 nor 2
8 UPSC CSE 15 MOCK TEST PAPERS GENERAL STUDIES PAPER I

75. Which of the following are the grounds of 80. “3D printing” has applications in which of the
disqualification under the Anti-defection law? following?
1. If any independently elected member joins any (1) Preparation of confectionery items
political party. (2) Manufacture of bionic ears
2. If any nominated member joins any political (3) Automotive industry
party after the expiry of six months. (4) Reconstructive surgeries
3. On being elected as the presiding officer of the (5) Data processing technologies
House, if a member voluntarily gives up the Select the correct answer using the code given below:
membership of his/her party or rejoins it after (a) 1, 3 and 4 only (b) 2, 3 and 5 only
he/she ceases to hold that office, he/she would (c) 1 and 4 only (d) 1, 2, 3, 4 and 5
be disqualified.
81. Consider the following statements regarding the
Which of the statements given above is/are correct?
Writ Jurisdiction of High courts:
(a) 1 only (b) 3 only
1. Writs Jurisdiction come under the ‘Original’
(c) 1 and 2 only (d) 1, 2 and 3
jurisdiction of the High Court.
76. Consider the following statements regarding the 2. The High Court can issue the writs only for the
Orang National park: enforcement of fundamental rights and not for
1. It is located on the bank of the Brahmaputra other purposes.
River in Arunachal Pradesh. Which of the above statements are correct?
2. A National Park is defined by Central government (a) 1 only (b) 2 only
via notification under the WPA. (c) Both 1 and 2 (d) Neither 1 nor 2
Which of the statements given above is/are correct?
82. A “Black Hole” is a body in space which does not
(a) 1 only (b) 2 only
allow any radiation to come out.
(c) Both 1 and 2 (d) Neither 1 nor 2
This property is due to its
77. Consider the following statements regarding Red (a) Very small size (b) Very large size
Sanders: (c) Very high density (d) Very low density
1. Red Sanders is an Indian endemic tree species
with a restricted geographical range in the 83. Consider the following statements:
western Ghats. 1. The Earth’s magnetic field has reversed every
2. It is under endangered category in IUCN Red few hundred thousand years.
List. 2. When the Earth was created more than 4000
Which of the statements given above is/are correct? million years ago there was 54% oxygen and no
(a) 1 only (b) 2 only carbon dioxide.
(c) Both 1 and 2 (d) Neither 1 nor 2 3. When living organisms originated, they modified
the early atmosphere of the Earth.
78. Consider the following statements regarding India
Which of the statements given above is/are correct?
State of Forest Report (ISFR) 2021:
(a) 1 only (b) 2 and 3 only
1. It is published every years by the Forest Survey
(c) 1 and 3 only (d) 1, 2 and 3
of India.
2. India’s total forest and tree cover is now more 84. Consider the following statements:
than 20% of the geographical area of the country. 1. On the planet Earth, the fresh water available for
3. Madhya Pradesh has the largest forest cover in use amounts to about less than 1% of the total
the country. water found.
Which of the statements given above is/are correct? 2. Of the total fresh water found on the planet Earth.
(a) 1 only (b) 2 and 3 only 95% is bound up in polar ice caps and glaciers.
(c) 1 and 2 only (d) 1, 2 and 3 Which of the statements given above is/ are correct?
(a) 1 only (b) 2 only
79. Assertion (A): Artificial satellites are always launched
(c) Both 1 and 2 (d) Neither 1 nor 2
from the earth in the eastward direction.
Reason (R): The earth rotates from west to east, and 85. The Nagara, the Dravida and the Vesara are the
so the satellite attains the escape velocity. (a) Three main tribal communities of the North east
(a) Both A and R are true, and R is the correct prevalent in India.
explanation of A. (b) Three main linguistic divisions into which the
(b) Both A and R are true, but R is NOT a correct languages of India can be classified.
explanation of A. (c) Three main styles of Indian temple architecture.
(c) A is true, but R is false. (d) Three main dance forms of the Indian sub-
(d) A is false, but R is true. continent.
MOCK TEST PAPER - 01 9

86. Consider the following statements regarding stocks: 93. Consider the following statement:
1. Preferred stock allows its holders to make a 1. Inflation benefits the debtors.
profit through rising share prices and dividend 2. Inflation benefits the bond holder.
payments. Which of the statements given above is /are correct?
2. Both common as well as preferred stock holders (a) 1 only (b) 2 only
have voting rights. (c) 1 and 2 only (d) Neither 1 nor 2
Which of the statements given above is/are correct? 94. Arrange the following in order of their increasing
(a) 1 only (b) 2 only liquidity.
(c) Both 1 and 2 (d) Neither 1 nor 2 1. Currency 2. Time deposit
3. Saving
87. Which of the following pairs is correctly matched? Select the correct answer using the codes given
(a) Diwan-i-Bandagani: Tughlaq below:
(b) Diwan-i-Mustakhraj: Balban (a) 1-3-2 (b) 2-3-1
(c) Diwan-i-Kohi: Alauddin Khalji (c) 3-1-2 (d) 1-2-3
(d) Diwan-i-Arz: Muhammad Tughlaq 95. The term marginal standing facility is used in a
88. Consider the following events: difference of which of the following sectors?
(1) Reign of Krishna Deva Raya of Vijayanagara (a) Banking
(2) Construction of Qutub Minar (b) Insurance
(3) Arrival of Portuguese in India (c) Infrastructure development
(4) Death of Feroz Tughlaq (d) Social sector
The correct chronological sequence of these 96. With reference to the term agreement on agriculture
events is: is used for which of the following organisation?
(a) 2-4-3-1 (b) 2-4-1-3 (a) WTO (b) IMF
(c) 4-2-1-3 (d) 4-2-3-1 (c) UNCTAD (d) ASEAN
97. Which of the following is correct when RBI is
89. During the time of which Mughal Emperor did
considering expansion in monetary policy?
the English East India Company establish its first
(a) Increasing the CRR
factory in India? (b) Increasing the SLR
(a) Akbar (b) Jahangir (c) Decreasing repo rate
(c) Shahjahan (d) Aurangzeb (d) Frequent use of open market operations
90. Consider the following statements: 98. Which of the following is part of non-planned
1. The Charter Act of 1853 abolished the East India expenditure?
Company’s monopoly of Indian trade. 1. Expenditure on defence
2. Under the Government of India Act 1858, the 2. Interest payment
British Parliament abolished the East India 3. Salaries
Company altogether and undertook the 4. Subsidies
responsibility of ruling India directly. Select the correct answer using the codes given
below:
Which of the statements given below is/are correct?
(a) 1, 2 and 3 only (B) 2 and 3 only
(a) 1 only (b) 2 only
(c) 1, 3 and 4 only (d) 1, 2, 3 and 4
(c) Both 1 and 2 (d) Neither 1 nor 2
99. With the decrease in the rate of interest of lending,
91. Which of the following organisations releases the which of the following option is correct?
World Economic Outlook report? (a) It will lead to the increase in investment
(a) World Economic Forum expenditure.
(b) Organisation of Economic Development and (b) Increase in the non-tax revenue of the
Cooperation government.
(c) International Monetary Fund (c) Will increase the foreign direct investment in the
(d) World Bank country.
(d) Increase the global export.
92. Which of the following is not a part of current
account? 100. Which of the following best explains the term
1. Foreign loan national income?
(a) Total value of goods and services produced by
2. Foreign Direct Investment
the nationals.
3. Private remittance
(d) Sum total of investment expenditure in the
4. Foreign Portfolio Investment
economy.
Select the correct answer using the codes given below: (c) Monetary value of the final goods and services
(a) 1, 2 and 3 only (b) 1, 2 and 4 only produced within a given period.
(c) 2 and 4 only (d) 4 only (d) Total cost of production.
10 UPSC CSE 15 MOCK TEST PAPERS GENERAL STUDIES PAPER I

ANSWERS WITH EXPLANATION


to Equality) and Fundamental Right 19 (Freedom of
1. Option (d) is correct.
Speech and Expression).
Explanation: Cases related to Preamble:
In the Golaknath Case (1967), the Supreme court had
OO Berubari v/s Union (1960) Case: Preamble is not pronounced that the parliament cannot amend the
part of the Constitution Fundamental Rights to give effect to the Directive
OO Kesavananda Bharati (1973) Case: Preamble is Principles of State Policy. The parliament responded
part of the Constitution by bringing the 25th Amendment Act (1971) of the
OO LIC of India (1995) Case: Preamble is an integral constitution which inserted Article 31C in Part III.
part of the Constitution. Article 31-C contained two provisions:
Note: Amendment of Preamble: Preamble can l If a law is made to give effect to DPSPs in Article
be amended without affecting basic features of 39(b) and Article 39(c) and in the process the law
the constitution. violates Article 14, Article 19 or Article 31, then
2. Option (d) is correct. the law should not be declared unconstitutional
and void merely on this ground.
Explanation: The eleven Fundamental Duties
l Any such law which contains the declaration
enshrined in Part IVA under Article 51A of the
that it is to give effect to DPSPs in Article 39(b)
constitution are:
& Article(c) shall not be questioned in a court of
l To abide with the Indian Constitution and
law.
respect its ideals and institutions such as the
National Anthem and Flag. 4. Option (c) is correct.
l To cherish and follow the noble ideas that Explanation: It was established in 1964 by the
inspired the national struggle for freedom. executive resolution of the central government.
l To protect the integrity, sovereignty and unity of But in 2003 Parliament gave it a statutory status by
India. enacting the Central Vigilance Commission (CVC)
l To defend the country and perform national Act 2003. It was established on the recommendation
services if and when the country requires. of the Santhanam Committee on Prevention of
l To promote the spirit of harmony and Corruption. Its jurisdiction extends to members of
brotherhood amongst all the people of India and All India Services serving in connection with the
renounce any practices that are derogatory to affairs of the Union Group-A officers of the Central
women. government, officers of the rank of Scale V and above
l To cherish and preserve the rich national heritage in Public Sector Banks etc.
of our composite culture.
5. Option (b) is correct.
l To protect and improve the natural environment
including lakes, wildlife, rivers, forests etc. Hence Explanation: The 7th Constitutional Amendment
statement 2 is correct. Act of 1956 facilitated the appointment of the same
l To develop scientific temper, humanism and person as a governor for two or more states. The
spirit of inquiry. 7th Amendment of the Indian Constitution was
l To safeguard all public property and to abjure needed to implement the recommendations of the
violence. States Reorganisation Commission regarding the
l To strive towards excellence in all genres of reorganization of the states on a linguistic basis.
individual and collective activities. Important provision of the 7th constitutional

l To provide opportunities for education to his/her amendment:
child or ward between the age of six and fourteen l A provision making it possible to appoint the
years. This fundamental duty was added by the same person as the Governor for two or more
86th Constitutional Amendment Act, 2002. States has been added to article 153.
l Articles 170 and 171 were amended. The
3. Option (c) is correct.
maximum strength of the Legislative Council of
Explanation: Article 39 (b) provides for the equitable a State has been raised from one-fourth to one-
distribution of material resources of the community third of the strength of the Legislative Assembly
for the common good, and Article 39 (c) provides of that State.
for the prevention of concentration of wealth and l Article 231 was amended to enable Parliament to
means of production. These two Directive Principles establish a common High Court for two or more
(under Article 39 (b) and Article 39 (c) thus have been States.
given precedence over Fundamental Right 14 (Right l Second and Seventh Schedules were amended.
MOCK TEST PAPER - 01 11

l Abolished the existing classification of states into 6. Option (d) is correct.


four categories i.e., Part A, Part B, Part C, and Part
D states, and reorganised them into 14 states and Explanation:
6 union territories. OO 86th Constitutional Amendment Act 2002 added
l Extended the jurisdiction of high courts to union the eleventh Fundamental duty to Part IV-A
territories. of the constitution under article 51-A, i.e., “To
provide opportunities for education to his child
l Provided for the establishment of a common
or ward between the age of six to fourteen years.
high court for two or more states.
OO 86th Constitutional Amendment Act added
l Provided for the appointment of additional and
the ‘Right to Education as a fundamental right
acting judges of the high court.
to Part III of the constitution under Article 21A
91st Constitutional Amendment Act, 2003:
which states that “The State shall provide free
Made the following provisions to limit the size of the and compulsory education to all children of the
Council of Ministers, to debar defectors from holding age of six to fourteen years in such manner as the
public offices and to strengthen the anti-defection State may by law determine.
law: OO 86th Constitutional Amendment Act also added
1. The total number of ministers, including the new article 45 to the constitution under Directive
Prime Minister, in the Central Council of Principles of State Policy which says “The state
Ministers shall not exceed 15% of the total shall endeavor to provide early childhood
strength of the Lok Sabha. care and education for all children until they
2. A member of either house of Parliament belonging complete the age of six years.
to any political party who is disqualified on the 7. Option (d) is correct.
ground of defection shall also be disqualified to
be appointed as a minister. Explanation: Articles 358 and 359 describe the
3. The total number of ministers, including the effect of a National Emergency on Fundamental
Chief Minister, in the Council of Ministers in a Rights. Article 358 deals with the suspension of
state shall not exceed 15% of the total strength the Fundamental Rights guaranteed by Article 19
and Article 359 deals with the suspension of other
of the legislative Assembly of that state. But,
Fundamental Rights except those guaranteed by
the number of ministers, including the Chief
Articles 20 and 21.
Minister, in a state shall not be less than 12.

Suspension of Fundamental Rights under Article 19:
4. A member of either House of a state legislature
According to Article 358, when a proclamation of
belonging to any political party who is
National Emergency is made, the six fundamental
disqualified on the ground of defection shall also rights under article 19 are automatically suspended.
be disqualified to be appointed as a minister. Article 19 is automatically revived after the expiry of
5. A member of either House of Parliament or either the emergency.
House of a State Legislature belonging to any l Article 19 containing six rights can be suspended
political party who is disqualified on the ground only when the National Emergency is declared
of defection shall also be disqualified to hold on the grounds of war or external aggression and
any remunerative political post. The expression not on the ground of armed rebellion.
“remunerative political post” means l This provision was added by the 44th
n any office under the central government constitutional amendment.
or a state government where the salary l Article 359 authorizes the president to suspend
or remuneration for such office is paid out the right to move any court for the enforcement
of the public revenue of the concerned of Fundamental Rights during a National
government; or Emergency.
n any office under a body, whether 8. Option (b) is correct.
incorporated or not, which is wholly or
Explanation: Part IV (article 36 to 51) of Indian
partially owned by the central government
constitution deals with the Directive Principles of
or a state government and the salary or
State Policy. Article 39(c) deals with prevention and
remuneration for such office is paid by
concentration of wealth. It says that “the operation
such body, except where such salary or
of the economic system does not result in the
remuneration paid is compensatory in
concentration of wealth and means of production to
nature.
the common detriment.”
6. The provision of the Tenth Schedule (anti-
defection law) pertaining to exemption from 9. Option (a) is correct.
disqualification in case of split by one-third Explanation: National Commission for Minorities:
members of legislature party has been deleted.
The National Commission for Minorities (NCM) was
It means that the defectors have no more set up under the National Commission for Minorities
protection on the grounds of splits. Act, 1992.
12 UPSC CSE 15 MOCK TEST PAPERS GENERAL STUDIES PAPER I

l NCM consists of a Chairperson, a Vice- n Enjoys privileges & immunities of a member


Chairperson and five members, and all of them of Parliament.
shall be from the minority communities. n Not a government servant & not debarred
l Total of 7 persons to be nominated by the Central from private legal practice.
Government should be from amongst persons l Limitations:
of eminence ability and integrity. Each member n No Right to Vote in House proceedings/
holds office for a period of three years from the committees.
date of assumption of office. n Should not advise against or hold a brief
l In 1993, the first Statutory National Commission
against the government.
was set up and five religious communities viz
n Should not defend accused persons
the Muslims Christians Sikhs Buddhists and
in criminal prosecutions without the
Zoroastrians (Parsis) were notified as minority
permission of the Government of India.
communities. In 2014, Jains were also notified
as a minority community. The NCM Act defines 12. Option (d) is correct.
a minority as “a community notified as such by Explanation: The Constitution prescribes imposition
the Central government.’’ Now, there are six of President’s Rule under Article 356 on two grounds-
religions, namely Muslims, Christians, Sikhs,
one mentioned in Article 356 itself and another in
Buddhists, Parsis (Zoroastrian) and Jain as
Article 365:
religious minorities in India.
l Article 356 empowers the President to issue a
10. Option (b) is correct. proclamation if he/she is satisfied that a situation
Explanation: The Public Accounts Committee was has arisen in which the government of a state
introduced in 1921 after its first mention in the cannot be carried on in accordance with the
Government of India Act 1919 also called Montford provisions of the Constitution. Notably, the
Reforms (Montagu–Chelmsford). president can act either on a report of the
l The PAC is formed every year with a strength of governor of the state or otherwise too (i.e., even
not more than 22 members of which 15 are from without the governor’s report).
Lok Sabha and 7 from Rajya Sabha. l Article 365 says that whenever a state fails to
l The term of office of the members is one year. comply with, or to give effect to any direction
l The Chairman is appointed by the Speaker from the Centre, it will be lawful for the
of Lok Sabha. Since 1967, the chairman of the president to hold that a situation has arisen in
committee is selected from the opposition. which the government of the state cannot be
l Its chief function is to examine the audit report
carried on in accordance with the provisions of
of the Comptroller and Auditor General (CAG)
the Constitution.
after it is laid in the Parliament.
13. Option (b) is correct.
11. Option (c) is correct.
Explanation: District Development Councils:
Explanation: Attorney General of India:

DDCs will act as a new unit of governance in J&K.

He/she is the legal officer in the country & part of
It will effectively replaces the District Planning and
Union Executive.
Development Boards in all districts and will prepare
l Assisted by 2 Solicitor Generals & 4 Additional
Solicitor Generals. and approve district plans and capital expenditure.
l Article 76 of constitution deals with for Attorney
Composition of DDCs:
General of India & Article 165 for Advocate l DDCs will have elected representatives from
General of States. each district. The term of the DDC will be five
l Eligibility: Qualified to be appointed as the years, and the electoral process will allow for
Supreme Court judge and has been a judge of reservations for Scheduled Castes, Scheduled
some high court for five years or an advocate Tribes and women.
of some high court for ten years or an eminent 14. Option (b) is correct.
jurist in the opinion of the President.
l Tenure: Not fixed. Explanation: Expenditure Limit in contesting
l Appointment is done by the President on the elections:
government advice.
The Election Commission of India imposes limits
l Rights and privileges on the expenditure incurred by a candidate but not
n Right to Speak & take part in proceedings of political parties on their election campaign.
both Houses of Parliament/joint sitting/any l The expenditure limits range from ` 20 lakh to
committee of Parliament of which he/she ` 28 lakh for assembly elections and from ` 54
may be named a member. lakh to ` 70 lakh for Lok Sabha elections.
MOCK TEST PAPER - 01 13

The expenditure limit was last revised in 2014


l OO Distribution of legislative powers between the
while the same was done for Andhra Pradesh Union and the states.
and Telangana in 2018 following their bifurcation OO Any of the lists in the Seventh Schedule.
in 2014. OO Representation of states in Parliament.
l After that, the limit has not been increased OO The Power of Parliament to amend the
despite an increase in the electorate and an Constitution and its procedure (Article 368 itself).
increase in the Cost Inflation Index. A Constitution Amendment Bill under article 368 can

So, recently the Election Commission of India (ECI) be introduced in either House of Parliament and has
has constituted a committee to examine the issues to be passed by each House by a special majority.
concerning the expenditure limit for a candidate. 17. Option (b) is correct.
15. Option (b) is correct. Explanation: Article 280 of the Constitution of India
Explanation: The Fundamental Rights are enshrined provides for a Finance Commission as a quasi-judicial
in Part III of the Constitution from Articles 12 to 35. body. It is constituted by the president of India every
Features of Fundamental Rights: fifth year or at such earlier time as he/she considers
necessary:
l They are not absolute but qualified. The state can
l The Finance Commission consists of a chairman
impose reasonable restrictions on them.
and four other members to be appointed by the
l They are not sacrosanct or permanent. The
president.
Parliament can curtail or repeal them but only
l They hold office for such period as specified by
by a constitutional amendment act and not by an the president in his order.
ordinary act. l They are eligible for reappointment.
l They are justiciable allowing persons to move l The Constitution authorizes the Parliament
the courts for their enforcement if and when to determine the qualifications of members of
they are violated. the commission and the manner in which they
l They are defended and guaranteed by the should be selected. For this purpose the Finance
Supreme Court. Hence, the aggrieved person Commission (Miscellaneous Provisions) Act 1951
can directly go to the Supreme Court. has been enacted.
l Most of them are available against the arbitrary The main functions of a finance commission are:
action of the State with a few exceptions like 1. The distribution of the net proceeds of taxes
those against the State’s action and against the between the centre and the states, and between
action of private individuals. For example, Article the states of the respective share of such
17, 23 and 24 provides protection against actions proceeds.
of private individuals. 2. The principles that should govern the grants-
l The Constitution under Article 32 and 226 in-aid to the states by the centre out of the
consolidated fund of India.
empowers the Supreme Court and High
3. Any matter referred to the commission by
Courts to issue writs for the enforcement of
the president of India in the interest of sound
Fundamental Rights. When the Fundamental
finance.
Rights of a citizen are violated the aggrieved
4. To supplement the resources of panchayats and
party has the option of moving either the high
the municipalities.
court or the Supreme Court directly. Further, the
18. Option (d) is correct.
Parliament (under Article 32) can empower any
other court to issue these writs. Explanation: The GST Council is a constitutional
Since no such provision has been made so far, only body under Article 279 A and was introduced by the
the Supreme Court and the high courts can issue the Constitution (One Hundred and First Amendment)
writs and not any other court. Act 2016.
l It makes recommendations to the Union and
16. Option (c) is correct.
State Government on issues related to Goods
Explanation: Constitutional amendment under and Service Tax.
Article 368 of the constitution requiring special Members of the GST Council includes:
majority for their passage and ratification by l Union Finance Minister (Chairperson of
Council).
Legislatures of not less than one-half of the States by
l Centre’s minister of state in-charge of revenue or
resolutions to that effect passed by those Legislatures.
Finance.
The following provisions can be amended in this l Minister of Revenue or Finance of all the states.
way: l The GST Council is chaired by the Union Finance
OO Election of the President and its manner. Minister and other members are the Union State
OO Extent of the executive power of the Union and Minister of Revenue or Finance and Ministers in-
the states, Supreme Court and high courts. charge of Finance or Taxation of all the States.
14 UPSC CSE 15 MOCK TEST PAPERS GENERAL STUDIES PAPER I

l It is considered a federal body where both the IIP remains extremely relevant for the calculation
centre and the states get due representation. of the quarterly and advance GDP (Gross
l Every decision of the Goods and Services Tax Domestic Product) estimates.
Council shall be taken at a meeting by a majority 22. Option (c) is correct.
of not less than three-fourths of the weighted
votes of the members present and voting. Explanation: AT-1 bonds are a type of unsecured
l The vote of the Central Government shall have perpetual bonds that banks issue to shore up their
a weightage of one third of the total votes cast, core capital base to meet the Basel-III norms.
and the votes of all the State Governments taken l There are two routes through which these bonds
together shall have a weightage of two-thirds of can be acquired:
the total votes cast, in that meeting n Initial private placement offers of AT-1 bonds
19. Option (b) is correct. by banks seeking to raise money.
n Secondary market buys of already-traded
Explanation: PM-KISAN: AT-1 bonds.

It was launched in February 2019. l These bonds offer higher returns to investors but
l It is a Central Sector Scheme with 100% funding carry a higher risk as well.
from the Government of India. l Investors cannot return these bonds to the
l Implemented by the Ministry of Agriculture and issuing bank and get the money. This means
Farmers Welfare. there is no put option available to its holders.
l Features: Under the scheme, the Centre transfers l Banks issuing AT-1 bonds can skip interest
an amount of ` 6,000 per year in three equal payouts for a particular year or even reduce the
instalments directly into the bank accounts of all bond’s face value provided their capital ratios
landholding farmers irrespective of the size of fall below certain threshold levels.
their land holdings. l If the RBI feels that a bank is on the brink of
l The entire responsibility of identification of collapse and needs a rescue, it can simply ask
beneficiary farmer families rests with the State/ the bank to cancel its outstanding AT-1 bonds
UT Governments. without consulting its investors.
20. Option (a) is correct. l Basel III is an internationally agreed set of
measures developed by the Basel Committee on
Explanation: Banking Supervision in response to the financial
l The Food Safety and Standards Authority of crisis of 2007-09.
India (FSSAI) is a statutory body established l The measures of Basel III aim to strengthen the
under the Food Safety and Standards Act 2006. regulation, supervision and risk management of
l The administrative ministry for FSSAI is the banks.
Ministry of Health & Family Welfare. It is l Basel is a city in Switzerland on the river Rhine.
responsible for protecting and promoting public
23. Option (b) is correct.
health through the regulation and supervision of
food safety. Explanation: A payments bank (Airtel Payments
Bank, India Post Payments Bank, etc.) is like any
21. Option (c) is correct.
other bank, but operating on a smaller or restricted
Explanation: Index of Industrial Production: scale. Credit risk is not involved with the Payments
l It is compiled and published monthly by → the Bank. It can carry out most banking operations
Central Statistical Organisation (CSO), Ministry but cannot advance loans or issue credit cards. It
of Statistics and Programme. can accept demand deposits only, i.e., savings and
l The Index of Industrial Production (IIP) is a current accounts, not time deposits.
composite indicator that measures changes in The Payment Banks cannot set up subsidiaries to
the volume of production of a basket of industrial undertake non-banking financial services activities.
products. Scope of Activities:
l Base Year: 2011-2012. l Acceptance of demand deposits, initially
l There are two ways in which IIP data can be restricted to holding a maximum balance of
viewed: ` 100,000 per individual customer.
n Broad sectors namely Mining Manufacturing l Issuance of ATM/debit cards.
and Electricity. l They cannot issue credit cards.
n Use-based sectors namely Basic Goods, l They are not allowed to give loans.
Capital Goods and Intermediate Goods. l Payments and remittance services through
l Significance of IIP: various channels.
It is used by government agencies including the l Distribution of non-risk sharing simple financial
Ministry of Finance, the Reserve Bank of India products like mutual fund units and insurance
etc, for policy-making purposes. products, etc.
MOCK TEST PAPER - 01 15

l They are only allowed to invest the money 26. Option (d) is correct.
received from customers’ deposits into
government securities. Explanation: National Bank for Agriculture and
l They cannot accept NRI deposits. Rural Development:
l A payments bank account holder would be able NABARD came into existence on 12th July 1982 by

to deposit and withdraw money through any transferring the agricultural credit functions of the
ATM or other service providers. Reserve Bank of India (RBI) and refinance functions
l Payments licensees would be granted to mobile of the then Agricultural Refinance and Development
firms, supermarket chains and others to cater to
Corporation.
individuals and small businesses.
l It is a statutory body established under the

The Payments Bank will be registered as a public
‘National Bank for Agriculture and Rural
limited company under the Companies Act, 2013.
Development Act 1981’.
It is governed by the provisions of the Banking
OO NABARD provides recommendations to RBI
Regulation Act, 1949; RBI Act, 1934; Foreign
Exchange Management Act, 1999, Payment and on the issue of licenses to Cooperative Banks,
Settlement Systems Act, 2007, other relevant Statutes opening of new branches by State Cooperative
and Directives. Banks and RRBs.
l They need to maintain a Cash Reserve Ratio l It is the apex banking institution to provide
(CRR). finance for Agriculture and rural development.
l Required to invest a minimum 75% of its “demand It supervises Cooperative Banks and Regional
deposit balances” in Statutory Liquidity Ratio Rural Banks (RRBs).
(SLR) eligible Government securities/treasury
27. Option (c) is correct.
bills with maturity up to one year.
l Need to hold maximum 25% in current and time/ Explanation: African Swine Fever is a highly
fixed deposits with other scheduled commercial contagious and fatal animal disease that infects and
banks for operational purposes and liquidity leads to an acute form of haemorrhagic fever in
management. domestic and wild pigs.
24. Option (c) is correct. l It was first detected in Africa in the 1920s.
Explanation: Indian Renewable Energy l It is caused by a large DNA virus of the
Development Agency: Asfarviridae family. It is not a threat to human

IREDA is a miniratna company under the Ministry of beings since it only spreads from animals to
New and Renewable Energy. other animals.
l It was set up in 1987 as a specialized non-banking l Mortality Rate is close to 100% and since the fever
finance agency for the renewable energy sector.
has no cure, the only way to stop it spreading is
l It is engaged in promoting, developing and
by culling the animals.
extending financial assistance for setting up
projects relating to new and renewable sources 28. Option (c) is correct.
of energy and energy efficiency/conservation. Explanation: Significance of Neem tree:
l Its motto is “ENERGY FOR EVER”.

Nearly all parts of the neem tree are useful. In
25. Option (c) is correct. many areas neem is considered a weed and based
Explanation: Foreign Contribution Regulation Act: on its antimicrobial resistance and other antifungal

This Act prohibits receipt of foreign funds by properties they are also used in pharmaceutical
candidates for elections, journalists or newspaper and cosmetic industries. Neem oil and neem barks
and media broadcast companies, judges and are used as an insect repellent. Neem is also used
government servants, members of legislature and
as the main component in some toothpaste and
political parties.
mouthwashes. Neem is commonly used in shampoos

It was enacted during emergency in 1976 in an
atmosphere of apprehension that foreign powers for treating dandruff and in soaps or creams for skin
were interfering in India’s affairs by pumping in allergies and infections. Neem leaves have been used
funds through independent organisations. as a traditional treatment for diabetes, and there is
l The law sought to regulate foreign donations some clinical evidence suggesting that it may help
to individuals and associations so that they control blood sugar levels. So, Neem tree has acquired
functioned “in a manner consistent with the industrial importance as a source of biofertilizer,
values of a sovereign democratic republic”. biopesticide and anti-fertility compound.
Once granted, FCRA registration is valid for five
29. Option (a) is correct.
years.
l It was amended in the year 2010 and then 2020. Explanation: Github is the world’s largest open-
l FCRA is implemented by the Ministry of Home source developer community platform where users
Affairs. upload their projects and code for others to view, edit
16 UPSC CSE 15 MOCK TEST PAPERS GENERAL STUDIES PAPER I

and tweak. The platform uses the software Git which OO It is not necessary that a person infected with HIV
was created in 2005 by Linus Trovalds, the developer will definitely develop AIDS. A person infected
of the open-source operating system Linux to track with HIV is likely to develop symptoms of AIDS
changes in a set of files and for coordination in over a period of time when his/her immune
software. Open source software (OSS) is software system is too weak to fight HIV infection.
that is distributed with its source code, making it OO HIV is transmitted from person to person
available for use, modification and distribution with through bodily fluids including blood, semen,
its original rights. vaginal secretions, anal fluids and breast milk.
30. Option (c) is correct. 32. Option (d) is correct.
Explanation: Explanation: India has started cultivation of Hing or
asafoetida for the first time. The first plantation was
done in Lahul and Spiti in Himachal Pradesh. It is
one of the widely used spices in Indian cuisine and
natural medicine. It is extracted from the fleshy roots
of perennial ferula (part of the celery family) as an
oleo-gum-resin.

The country imports about 1540 tonnes of raw
asafoetida annually from Afghanistan, Iran and
Uzbekistan and spends approximately `942 crore per
year on it. It is important for India to become self-
sufficient in asafoetida production.
33. Option (a) is correct.
Explanation: CITES (the Convention on
International Trade in Endangered Species of Wild
Fauna and Flora) is an international agreement
between governments. Its aim is to ensure that
international trade in specimens of wild animals and
plants does not threaten their survival.

CITES was drafted as a result of a resolution adopted
in 1963 at a meeting of members of IUCN (The World
Figure: NISAR mission
Conservation Union).

NISAR is a joint Earth-observing mission between

CITES is an international agreement to which States
NASA and the Indian Space Research Organization and regional economic integration organizations
(ISRO). adhere voluntarily. States which have agreed to be
l This satellite will be able to detect the earth’s bound by the Convention (‘joined’ CITES) are known
surface movement as small as 0.4 inches over an as Parties. Although CITES is legally binding on the
area. parties-in other words, they have to implement the
Convention-it does not take the place of national
l The name NISAR is short for NASA-ISRO-SAR.
laws. Rather, it provides a framework to be respected

 SAR refers to the Synthetic Aperture Radar by each party, which has to adopt its own domestic
that NASA will use to measure changes in the legislation to ensure that CITES is implemented at
surface of the Earth. It refers to a technique for the national level.
producing high-resolution images. It will scan 34. Option (b) is correct.
the globe every 12 days over the course of its
Explanation:
three-year mission of imaging the Earth’s land. Interspecific interactions arise from the interaction of
OO NASA will provide one of the radars for the populations of two different species. They could be
satellite, a high-rate communication subsystem beneficial, detrimental or neutral to one of the species
for science data GPS receivers and a payload or both. Both the species benefit in mutualism and
data subsystem. both lose in competition in their interactions with
OO ISRO will provide the spacecraft bus, the second each other. Lichens result from the symbiotic union
between fungi and algae or fungi and cyanobacteria.
type of radar (called the S-band radar) the launch
The fungus receives nutrients obtained from the
vehicle and associated launch services.
photosynthetic algae or bacteria while the algae or
31. Option (a) is correct.
bacteria receive food protection and stability from
Explanation: The ELISA test is widely used to test the fungus. Clownfish and sea anemones have a
for HIV antibodies. mutualism relationship in which each party provides
MOCK TEST PAPER - 01 17

valuable services for the other. Clownfish live within 37. Option (d) is correct.
the protective tentacles of the sea anemone. In return,
the sea anemone receives cleaning and protection. Explanation: Recently the tiger survey has

In both parasitism and predation only one species highlighted that the tiger population in the source-
benefits (parasite and predator respectively) and the sink is in the ratio of 60:40.
interaction is detrimental to the other species (host
Source-sink dynamics is a theoretical model used
and prey respectively). The best-known examples by ecologists to describe how variation in habitat
quality may affect the population growth or decline
of predation involve carnivorous interactions in
of organisms. In this model, organisms occupy two
which one animal consumes another such as tiger
eating deer. Carnivorous plants such as the Venus patches of habitat.
fly trap and the pitcher plant consume insects are l Source is a high-quality habitat that on average
also examples of Predation. The interaction where allows the population to increase.
one species is benefitted, and the other is neither l Sink is a very low-quality habitat that on its own
benefitted nor harmed is called commensalism. would not be able to support a population.
In amensalism on the other hand, one species is
However, if the excess of individuals produced in
harmed whereas the other is unaffected. One of the source frequently moves to the sink, the sink
the best-known examples of commensalism is the population can persist indefinitely.
remora (family Echineidae) that rides attached to
33% of the tiger population in India lives outside its
sharks and other fishes. Predation parasitism and source, i.e., tiger reserves.
commensalism share a common characteristic– the
17 out of 50 tiger reserves in India are going to achieve
interacting species live closely together.
its maximum capacity to hold the tiger populations.
35. Option (c) is correct.
The Tiger survey suggested that there is a need to
Explanation: A study has pointed out that Karnataka, create buffer areas around the habitat zones of tigers
Maharashtra and Madhya Pradesh rank high in the where guided land-use and faster conservation
conservation of the endangered Dhole in India. The interventions can help reduce human-tiger conflict.
Dhole is a canid (a mammal of the dog family) native 38. Option (c) is correct.
to Central South and Southeast Asia. Other names
for the species include Asiatic wild dog, Indian wild Explanation: International Solar Alliance:
dog, whistling dog, red dog and mountain wolf. It is a multilateral forum jointly launched by India
The dhole is a highly social animal living in large and France on the side-lines of the 21st Conference
clans without rigid dominance hierarchies and of Parties (COP 21) to the United Nations Framework
containing multiple breeding females. Convention on Climate Change in 2015. Its aim is
l Endangered – IUCN to work for efficient exploitation of solar energy to
l Schedule II of wildlife act reduce dependence on fossil fuels.
l CITES – Appendix II Objectives:
Threats: Depletion of prey base Habitat loss and l To collectively address key common challenges
transformation Persecution of Dholes stems mainly
to scale up solar energy applications in line with
from retaliatory killings due to livestock predation
their needs.
Competition with other species like Tigers and
l To mobilize investments of more than USD 1000
Leopards for prey.
billion by 2030.
36. Option (b) is correct.
l To bring about a major decrease in the cost of
Explanation: The coral is a polyp, an organism that solar energy.
lives in the shallow sea. Its skeleton is composed of l To scale up applications of solar technologies in
limestone and dolomite. The layers of deposition of
member countries and facilitate collaborative
the skeletons of these polyps form a shallow rock
known as Coral Reef. research and development (R&D).

They thrive in tropical oceans confined between 25° The International Solar Alliance (ISA) is an alliance of
N and 25°S latitudes. Corals are found mainly in the more than 122 countries initiated by India.
tropical oceans and seas because they require a high Headquarter: Gurugram India.
mean annual temperature above 20° Celsius. The alliance is a treaty-based intergovernmental

Most fishes lay eggs in coral colonies. Since coral
organisation. Countries that do not fall within the
polyps cannot survive above water level, coral reefs
are found either up to sea level or below it. tropics can join the alliance and enjoy all benefits as

The coral reefs are more diverse than tropical other members with the exception of voting rights.
rainforests because coral reefs have more than After the United Nations it is the largest grouping of
1000000 species. states world-wide.
18 UPSC CSE 15 MOCK TEST PAPERS GENERAL STUDIES PAPER I

39. Option (a) is correct. 42. Option (c) is correct.


Explanation: Recently, IIT Kanpur has developed Explanation:
indigenous seed balls named BEEG (Bio-compost OO Pushyamitra Sunga was the founder of the
Enriched Eco-friendly Globule) which will help Sunga Dynasty. He was a Brahmin and army
farmers in the plantation with safety in COVID chief under Brihadratha, the last Mauryan King.
times. These are seed balls consisting of indigenous n He defeated Brihadratha in an internal revolt
variety of seeds, compost and clay. There is no need and established the Sunga Empire in 180 BC
for digging pits for planting saplings. These seed with its capital at Pataliputra.
balls are to be thrown at aimed places, and they will OO Devabhuti was the last king of the Shunga
germinate when come into contact with water. BEEG Empire. He was killed by Vasudeva Kanva.
is enriched with the right ingredients and seeds n The Kanva dynasty was founded by
to germinate early, and are the best way to utilise Vasudeva Kanva.
monsoon and plant as many trees as possible without OO Simuka was the founder of the Satavahana
risking lives by social gathering during Covid-19. Dynasty. Simuka is mentioned as the first king
40. Option (c) is correct. in a list of royals in a Satavahana inscription at
Naneghat.
Explanation: The Union Environment Minister
OO Chandragupta Vikramaditya was the founder of
has launched the Surakhsya Portal on the World
the Gupta dynasty.
Elephant Day. It is the National Portal on human
elephant conflict. 43. Option (b) is correct.
It is for the collection of real-time information & Explanation:
also for managing the conflicts on a real-time basis. OO Sultan Raziyyat-Ud-Dunya Wa Ud-Din,
The World Elephant Day is observed on August 12 popularly known as Razia Sultana, was a ruler
to create awareness of the urgent plight of African of the Delhi Sultanate. She was the first female
and Asian elephants and to share the knowledge and Muslim ruler of the subcontinent, and the only
positive solutions for the better care and management female Muslim ruler of Delhi.
of captive and wild elephants. n She ruled the court of Delhi from the end of
Conservation Status: The African elephants are listed 1236 to 1240.
as Vulnerable and Asian elephants as Endangered on OO Iltutmish was the third of the Mamluk kings who
the IUCN Red List of threatened species. ruled the former Ghurid territories in northern
The Project Elephant (PE) was launched by the
India.
Government of India in 1992 as a Centrally Sponsored
n He was the first Muslim sovereign to rule
Scheme to:
from Delhi, and is thus considered the
(a) Protect elephants their habitat & corridors,
effective founder of the Delhi Sultanate.
(b) Address issues of man-animal conflict, and
n His regime was between 1211 to 1236.
(c) Welfare of captive elephants.
OO Akbar was the third Mughal emperor, who
41. Option (c) is correct. reigned from 1556 to 1605.
Explanation: TRAFFIC India has released a paper n Akbar succeeded his father, Humayun,
titled ‘‘SPOTTED” in ‘Illegal Wildlife Trade: A Peek under a regent, Bairam Khan, who helped
into Ongoing Poaching and Illegal Trade of Leopards the young emperor expand and consolidate
in India’. Of the total of 747 leopard deaths between Mughal domains in India.
2015-2019 in India, 596 were linked to illegal wildlife OO Ibrahim Khan Lodhi, was the last Sultan of the
trade and activities related to poaching. The highest Delhi Sultanate, who became Sultan in 1517 after
numbers of poaching incidents were reported from the death of his father Sikandar Khan Lodi.
the States of Uttarakhand and Maharashtra. TRAFFIC n He became Sultan in 1517 after the death of
is the leading non-governmental organisation his father Sikandar Khan Lodi.
(NGO) working globally on the trade of wild animals n He was the last ruler of the Lodi dynasty,
and plants in the context of both biodiversity and reigning for nine years until 1526.
sustainable development. It was founded in 1976 n He was defeated and killed at the Battle of
as a strategic alliance of the World Wide Fund for Panipat by Babur's invading army, giving
Nature (WWF) and the International Union for the way to the emergence of the Mughal Empire
Conservation of Nature (IUCN). The Indian leopard in India.
is a leopard subspecies widely distributed on the 44. Option (C) is correct.
Indian subcontinent.
l It is distributed in India, Nepal, Bhutan and parts Explanation:
of Pakistan. OO Maitreya is a transcendent bodhisattva named as
l IUCN Status: Vulnerable the universal Buddha of a future time.
l Threats: Habitat Destruction Human-Animal n The name is taken from the Sanskrit maitri (in
Conflict and Illegal Wildlife Trade. Pali, metta), which means "loving kindness."
MOCK TEST PAPER - 01 19

n In Mahayana Buddhism, Maitreya is the Explanation:


embodiment of all-encompassing love. Mahajanapadas were sixteen kingdoms that
OO Amitayus, the Buddha of Eternal Life, is also existed in the Northern ancient India from the
known as Amitabha, one of the five Cosmic 6th to 14th centuries BCE.
Buddhas of Esoteric Buddhism. Magadha became the most powerful
n He is shown in his paradise, Sukhavati, the mahajanapada among the 16 Mahajanapadas
Western Pure Land, enthroned beneath a due to -
flowering tree festooned with strands of n Magadha was a region where agriculture
jewels and auspicious symbols. was productive.
OO Avalokiteshvara is the earthly manifestation of n Besides, iron mines were accessible and
the self-born eternal Buddha Amitabha, whose provided resources for tools and weapons.
figure is represented in his headdress. n Elephants were an important component of
n He guards the world in the interval between the army, were found in the forests region.
the departure of the historical Buddha, n The Ganga and its tributaries provided
Gautama, and the appearance of the future a means of cheap and convenient
buddha, Maitreya. communication.
n Padamapani is another name in Sanskrit for
Initially, Rajagriha was the capital of Magadha.
Bodhisattva Avalokitesvara, who represents
Rajagriha was a fortified settlement located amongst
the compassion of all of the Buddhas hills. Later in the fourth century BCE the capital was
45. Option (b) is correct. shifted to Pataliputra, present-day Patna.
Explanation: Indus people produced wheat barley, 47. Option (b) is correct.
peas, sesame, mustard and rice. Food grains were Explanation: During the Vedic age the king‘s
stored in huge granaries in both Mohenjodaro and influence was strengthened by various rituals like.
Harappa. The Harappans probably used the wooden Rajasuya Sacrifice: It is a king‘s inaugural sacrifice.
ploughshare. The Indus people were the earliest
After conquering the kings of several other kingdoms
people to produce cotton. Ragi or finger millet is not
and collecting tribute from the conquered land, the
known so far to any of the Harappan sites in north
India. vanquished kings are invited to attend the Rajasuya
yajna. All the vanquished kings would consider the
46. Option (d) is correct.
performer of this yajna as their Emperor.
Vajapeya Sacrifice: Chariot race in which a royal
chariot was made to win the race against his kinsmen.
Ashvamedha Sacrifice: Unquestioned control over
the area on which the royal horse ran uninterrupted.
48. Option (a) is correct.
Explanation: It is one of the classical dances from the
state of Kerala.

It gets its name from the word Mohini – a historical
enchantress avatar of the Hindu god Vishnu who
helps the good prevail over evil by developing
her feminine powers. It traces its roots from the
Natya Shastra, the ancient Hindu Sanskrit text on
performance arts.

It is traditionally a solo dance performed by women
after extensive training.
49. Option (b) is correct.
Explanation: Jataka stories are literature concerning
the previous births of Buddha in both human and
Sixtee Mahajanapadas
animal form. Jataka stories were depicted on the
railings and toranas of the stupas.

The Jataka stories that find depiction frequently
are Chhadanta Jataka, Sibi Jataka, Ruru Jataka,
This map is not to scale and is for reference only. Vessantara Jataka, Vidur Jataka and Shama Jataka.

20 UPSC CSE 15 MOCK TEST PAPERS GENERAL STUDIES PAPER I

50. Option (c) is correct. 51. Option (d) is correct.


Explanation: The Portuguese were the first
European who landed at Pondicherry. The name
is recorded in its Portuguese version as Puducheria
for the first time in the map of India dated 1554. The
Portuguese settlement of Puducheria continued till
1614. Fort St. George is the first English fortress in
India, founded in 1644, at the coastal city of Madras,
the modern city of Chennai. Fort William is a fort in
Calcutta (Kolkata) built during the early years of the
Bengal Presidency of British India.
52. Option (b) is correct.
Explanation: Sequence of the establishment of
European Companies:
(1) Portuguese (1498)
(2) English East India Company (1600)
(3) Dutch East India Company (1602)
(4) Danish East India Company (1616)
(5) French East India Company (1664)
53. Option (d) is correct.
Explanation:
OO The Battle of Madras or Fall of Madras took
place in September 1746 during the War of
the Austrian Succession when a French force
attacked and captured the city of Madras from its
British garrison under the leadership of Joseph
François Dupleix.
OO Battle of Plassey (1757): Participants in the War:
The British East India Company commanded by
Robert Clive against Siraj-ud-Daulah, the Nawab
of Bengal Bihar and Orissa and a small French
force.
OO Third Battle of Panipat (1761): Participants in the
War: Between a northern expeditionary force
Figure: Kathak of the Maratha Empire and a coalition of the
Explanation: Kathak is one of the eight major forms King of Afghanistan, Ahmad Shah Durrani, the
of Indian classical dance. Its origin is traditionally Rohilla Afghans of the doab and Shuja-ud-Daula
attributed to the travelling bards in the north of the Nawab of Oudh.
Ancient India known as Kathakars or storytellers. OO Battle of Buxar (1764): Fought between the
Kathak evolved during the Bhakti movement British East India Company and Mir Qasim the
particularly by incorporating the childhood and Nawab of Bengal, Shuja-ud-Daulah the Nawab
stories of the Hindu god Krishna. It is the only of Awadh and Shah Alam II the Mughal Emperor.
classical dance of India having links with Muslim
culture. It represents a unique synthesis of Hindu 54. Option (c) is correct.
and Muslim genius in art.

The 8 Indian classical dance forms or traditional Explanation:
Indian dances recognised by Sangeet Natak Academy OO The Third Battle of Panipat was fought between
are: the Afghan forces of Ahmad Shah Durrani along
1. Bharatnatyam (Tamil Nadu) with his local Rohilla and other Pathan and
2. Kathak (Uttar Pradesh) Oudh allies against the Maratha Empire.
3. Kuchipudi (Andhra Pradesh)
4. Odissi (Odisha) OO In the Third Anglo-Mysore War (1790–92),
5. Kathakali (Kerala) Tipu Sultan, the ruler of Mysore formed an
6. Sattriya (Assam) ally with France and invaded the nearby state
7. Manipuri (Manipur) of Travancore in 1789 which was a British ally.
8. Mohiniyattam (Kerala) The British forces were commanded by the
MOCK TEST PAPER - 01 21

Governor-General Cornwallis himself. The 58. Option (c) is correct.


resultant war lasted three years and was a defeat
for Mysore. The war ended after the 1792 Siege Explanation: The Cabinet Mission included Lord
of Seringapatam and the signing of the Treaty of Pethick Lawrence, the Secretary of State for India, Sir
Stafford Cripps and A. V. Alexander.
Seringapatam according to which Tipu had to

The Cabinet Mission recommended an undivided
surrender half of his kingdom to the British East
India and turned down the Muslim league’s demand
India Company and its allies.
for a separate Pakistan.
OO The Battle of Plassey was a decisive victory of the

The existing provincial assemblies were proposed to
British East India Company over the Nawab of
be grouped into three sections:
Bengal Siraj-ud-Daulah and his French allies in
OO Section A comprising Madras, Bombay, Uttar
1757 under the leadership of Robert Clive which
Pradesh, Bihar, Chattisgarh and Odisha
was possible due to the defection of Mir Jafur Ali OO Section B consisting of Punjab, NWFP and
Khan who was Siraj-ud-Daulah’s commander in OO Section C comprising Bengal and Assam.
chief.
It provided that the central government at Delhi
55. Option (c) is correct. would have powers over defence, foreign affairs and
communications. The rest of the powers would be
Explanation: Tipu Sultan established embassies with
vested with the provinces.
many foreign countries like France, Pegu, Turkey,
Iran etc. 59. Option (c) is correct.

The Nawabs of Oudh were the faithful allies of the Explanation: The Brahmo Sabha was founded by
English. Dalhousie was determined to annex Oudh the Hindu reformer Ram Mohan Roy (1772-1833) in
on whatever ground. At last, towards the close of his August 1828. Raja Rammohan Roy was involved in
rule he brought serious charges against the ruling various efforts to spread education in India. In 1825
of Nawab Wazid Ali that his administration had he founded the Vedanta college.
become a complete misrule. The Nawab was forced
Dayanand Saraswati established the Arya Samaj at
to vacate his throne and Dalhousie annexed Oudh Bombay on 10th April 1875, and later the headquarters
on February 1856. This annexation of Oudh was an of the Arya Samaj were established at Lahore.
example of reckless imperialism.
60. Option (d) is correct.
56. Option (c) is correct.
Explanation: The Umngot is a river in Meghalaya.
Explanation: The Gandhi-Irwin Pact of 1931 was It is considered to be India’s cleanest river. The
signed by Gandhiji on behalf of the Congress and by river attracts many tourists to Dawki bordering
Lord Irwin on behalf of the Government. Bangladesh. The river is the natural boundary

The terms of the agreement included the immediate between Ri Pnar (of Jaintia Hills) with Hima Khyrim
release of all political prisoners not convicted for
(of Khasi Hills). Dawki Bridge is a suspension bridge
violence, the remission of all fines not yet collected,
over the Umngot River.
the return of confiscated lands not yet sold to third
parties and lenient treatment for those government 61. Option (c) is correct.
employees who had resigned.
l The Government also conceded the right to
make salt for consumption to villages along the
coast and also the right to peaceful and non-
aggressive picketing.
l The Congress demand for a public inquiry into
police excesses was not accepted.
57. Option (a) is correct. Ivory
Coast
ia
al

Explanation: The Simon Commission was appointed


m
So

to go into the question of further constitutional


reform for British India. All the members of the
Commission were Englishmen. This announcement
was greeted by a chorus of protest from all Indians.
The Indian National Congress decided to boycott the Zimbabwe

Commission ‘at every stage and in every form.’ The


Muslim League and the Hindu Mahasabha decided
to support the Congress decision. ‘Go Back Simon’
was the famous slogan at that time. The Simon
Commission report contained no mention of the This map is not to scale and is for reference only.
Dominion Status. Figure: Map of Africa
22 UPSC CSE 15 MOCK TEST PAPERS GENERAL STUDIES PAPER I

Explanation: Tanzania is a country in East Africa 64. Option (a) is correct.


within the African Great Lakes region. It borders
Uganda to the north; Kenya to the northeast; Explanation: New Development Bank:
Comoro Islands and the Indian Ocean to the east; It is a multilateral development bank operated by the
Mozambique and Malawi to the south; Zambia BRICS states (Brazil, Russia, India, China and South
to the southwest; and Rwanda Burundi and the Africa).
Democratic Republic of the Congo to the west. It was established in 2014 at the 6th BRICS Summit at

Mount Kilimanjaro, Africa’s highest mountain, is Fortaleza, Brazil.
located in Tanzania. Dodoma is the capital of the The bank is set up to foster greater financial and
country. development cooperation among the five emerging
62. Option (d) is correct. markets.
Headquarter: Shanghai, China.
Unlike the World Bank, which assigns votes based on
capital share, in the New Development Bank, each
participant country will be assigned one vote and
none of the countries will have veto power.
In 2018 the NDB received observer status in the
United Nations General Assembly.
65. Option (b) is correct.
Explanation: North Atlantic Treaty Organization:

It is an intergovernmental military alliance
established by the North Atlantic Treaty (also called
the Washington Treaty) of April 4th 1949 by the
United States, Canada and several Western European
nations.
Headquarters: Brussels, Belgium.
Its purpose is to guarantee the freedom and security
of its members through political and military means.
This map is not to scale and is for reference only.
It promotes democratic values and enables members
Figure: Kowloon Peninsula to consult and cooperate on defence and security-
related issues to solve problems, build trust and in
Explanation: The Kowloon Peninsula is a peninsula the long run prevent conflict. It is committed to the
that forms the southern part of the main landmass peaceful resolution of disputes. If diplomatic efforts
in the territory of Hong Kong alongside Victoria fail, it has the military power to undertake crisis-
Harbour and facing toward Hong Kong island. management operations.

It constitutes a system of collective defence whereby

Geographically, the term “Kowloon Peninsula” may
its independent member states agree to mutual
also refer to the area south of the mountain ranges of defence in response to an attack by any external
Beacon Hill, Lion Rock, Tate’s Cairn, Kowloon Peak party. Since its founding, the admission of new
etc. member states has increased the alliance from the
original 12 countries to 30. The most recent member
63. Option (c) is correct.
state to be added to NATO was North Macedonia on
Explanation: The Godavari is the largest Peninsular 27th March 2020.
river system. It is also called the Dakshin Ganga. 66. Option (d) is correct.

The Godavari river rises from Trimbakeshwar near
Explanation: Project DANTAK was established
Nasik in Maharashtra and flows for a length of about
in 1961 under the leadership of the Third King of
1465 km before falling into the Bay of Bengal. Bhutan and then Indian Prime Minister Jawahar Lal
l The Godavari basin extends over the states Nehru.
of Maharashtra, Telangana, Andhra Pradesh, l It was established with the objective of
Chhattisgarh and Odisha in addition to smaller developing roads, telecommunication networks
parts in Madhya Pradesh, Karnataka and the and other such landmark infrastructure-related
Union territory of Puducherry. projects in Bhutan.
l It’s tributaries are Pravara, Purna, Manjra, l The Border Roads Organisation (BRO) was the
Penganga, Wardha, Wainganga, Pranhita nodal agency.
(combined flow of Wainganga, Penganga and 67. Option (d) is correct.
Wardha) Indravati, Maner and the Sabri. Explanation: Article 356 of the Constitution of India
l Kumbh Mela also takes place on the banks of the empowers the President to withdraw from the Union
Godavari river in Nashik. the executive and legislative powers of any state “if
MOCK TEST PAPER - 01 23

he is satisfied that a situation has arisen in which l The Index ranks 180 countries and regions
the government of the state cannot be carried on in according to the level of freedom available to
accordance with the provisions of the Constitution. journalists. The index ranks 180 countries based

A proclamation imposing President’s Rule must be on the following parameters such as:
approved by both the Houses of Parliament within (a) pluralism
two months from the date of its issue: (b) media independence
l However, if the proclamation of President’s Rule (c) environment and self-censorship
is issued at a time when the Lok Sabha has been (d) legislative framework
dissolved or the dissolution of the Lok Sabha (e) Transparency
takes place during the period of two months (f) Infrastructure and
without approving the proclamation, then the (g) Abuses.
proclamation survives until 30 days from the first Norway tops the index followed by Finland and
sitting of the Lok Sabha after its reconstitution Denmark.
provided the Rajya Sabha approves it in the l India ranks 142nd on the World Press Freedom
meantime. If approved by both the Houses of Index 2021. The rank is the same as in 2020.
Parliament, the President’s Rule continues for l Nepal is at 106, Sri Lanka at 127, Myanmar (before
six months. It can be extended for a maximum the coup) at 140, Pakistan at 145 and Bangladesh
period of three years with the approval of the at 152.
Parliament every six months. 71. Option (b) is correct.
68. Option (c) is correct. Explanation: The Siachen Glacier is a glacier located
Explanation: Gold Exchange would be a national in the eastern Karakoram range in the Himalayas
platform for buying and selling EGRs (Electronic just northeast of the point NJ 9842 where the Line of
Gold Receipts) issued against physical gold. Control between India and Pakistan ends.
Investors can trade in EGRs on stock exchanges and l The glacier lies between the Saltoro Ridge
the proposed gold exchange. immediately to the west and the main Karakoram
l The transaction in a gold exchange has been range to the east.
divided into three parts-conversion of physical l The Saltoro Ridge originates in the north from
gold into EGR, trading of EGR on a stock the Sia Kangri peak on the China border in the
exchange and conversion of EGR into physical Karakoram range.
gold. l The entire Siachen Glacier with all major passes is
currently under the administration of India since
l SEBI (Securities and Exchange Board of India)
1984 (Operation Meghdoot). Pakistan controls
would regulate the entire ecosystem of the
the region west of Saltoro Ridge far away from
proposed gold exchange. It would be the
the glacier.
sole regulator for the exchange, including

The glacier’s melting waters are the main source of
for vaulting, assaying gold quality and fixing
the Nubra River in the Indian region of Ladakh.
delivery standards. Physical gold deposited at
one location can be withdrawn from a different 72. Option (d) is correct.
location of any vault manager. Explanation: Indonesia’s parliament passed a law
69. Option (b) is correct. approving the relocation of its capital from slowly
sinking Jakarta to a site 2000 kilometres away on
Explanation: Khajuraho Temples (in Madhya the jungle-clad Borneo island that will be named
Pradesh) are among the most beautiful medieval “Nusantara”. Indonesia is not the first country in
monuments in the country. the region to relocate from an overpopulated capital.
l It is built between 950-1050 AD by the Chandela Malaysia moved its government to Putrajaya from
Dynasty. Kuala Lumpur in 2003 while Myanmar moved its
l The monuments include Hindu and Jain temples. capital to Nyaypyidaw from Rangoon in 2006.
l The temple site is situated within the Vindhya
73. Option (b) is correct.
Mountain range.
l The temples are famous for their Nagara-style Explanation: Article 262 provides for the adjudication
architectural symbolism. of inter-state water disputes. It has two provisions:
l These Temples got the status of UNESCO’s OO Parliament may by law provide for the
World Heritage Sites in 1986. adjudication of any dispute or complaint with
respect to the use, distribution and control of
70. Option (b) is correct.
waters of any inter-state river and river valley.
Explanation: India has ranked 142nd out of 180 OO Parliament may also provide that neither the
nations in the World Press Freedom Index. Supreme Court nor any other court is to exercise

It is published by the Reporters Sans Frontiers (RSF) jurisdiction in respect of any such dispute or
or Reporters Without Borders since 2002. complaint.
24 UPSC CSE 15 MOCK TEST PAPERS GENERAL STUDIES PAPER I

Under the provisions of the act, the central


l OO If any independently elected member joins any
government has enacted River Boards Act (1956) political party.
and Inter-state Water Disputes Act (1956). OO If any nominated member joins any political

The Inter-state Water Disputes Act empowers the party after the expiry of six months.
central government to set up an ad hoc tribunal for l Exceptions under anti-defection law:
The law allows a party to merge with or into
the adjudication of a dispute between the two or
another party provided that at least two-thirds
more states in relation to the water of an inter-state
of its legislators are in favour of the merger.
river. The decision of the tribunal would be final and On being elected as the presiding officer of the
binding. The act bars the SC and any other court to House, if a member voluntarily gives up the
have jurisdiction in this matter. membership of his/her party or rejoins it after
74. Option (c) is correct. he/she ceases to hold that office, he/she won’t be
disqualified.
Explanation: The law was originally drafted in 1837
by Thomas Macaulay, the British historian-politician, 76. Option (d) is correct.
but was inexplicably omitted when the Indian Penal Explanation: Orang National park is located on
Code (IPC) was enacted in 1860. the north bank of the Brahmaputra River in the
l Section 124A was inserted in 1870 by an
Darrang and Sonitpur districts of Assam, India. It
amendment, Sedition Law today is a crime under
Section 124A of the Indian Penal Code (IPC). was established as a sanctuary in 1985 and declared
OO Section 124A IPC → It defines sedition as a National Park on 13 April 1999. It has a rich flora
an offence committed when “any person by and fauna including the great Indian rhinoceros,
words either spoken or written or by signs or pygmy hog, Asian elephant, wild water buffalo and
by visible representation or otherwise brings Bengal tiger. A National Park is defined by the state
or attempts to bring into hatred or contempt or government via notification under the WPA (Wildlife
excites or attempts to excite disaffection towards
Protection Act 1972):
the government established by law in India”.
l The state government can fix and alter boundaries
Disaffection includes disloyalty and all feelings
of enmity. However, comments without exciting of the National Parks with prior consultation and
or attempting to excite hatred, contempt or approval with the National Board of Wildlife.
disaffection will not constitute an offence under
There is no need to pass an act for the alternation
this section. of boundaries of National Parks. No human activities
l The offence of Sedition is a non-bailable offence. are permitted in a National Park.
Punishment under the Section 124 A ranges from 77. Option (b) is correct.
imprisonment up to three years to a life term to
which fine may be added. A person charged Explanation: Red Sanders is an Indian endemic tree
under this law is barred from a government job. species with a restricted geographical range in the
They have to live without their passport and Eastern Ghats.
must produce themselves in the court at all times l The species is endemic to a distinct tract of forests
as and when required. in Andhra Pradesh. Some contiguous patches in
l The Supreme Court on 12 May 2022 put sedition Tamil Nadu and Karnataka.
law on hold and said 'no new case till further l Red Sanders usually grow in the rocky degraded
orders'. and fallow lands with Red Soil, and hot and dry
75. Option (c) is correct. climate.
Protection Status of Red Sanders
Explanation: The anti-defection law was passed in
1985 through the 52nd Constitutional Amendment IUCN Red List-Endangered.
Act. It added the Tenth Schedule to the Indian CITES-Appendix II
Constitution. Wildlife (Protection) Act 1972 -Schedule II.
Grounds of Disqualification: 78. Option (b) is correct.
OO If an elected member voluntarily gives up his/her
membership of a political party. Explanation:
OO If he/she votes or abstains from voting in such OO India State of Forest Report (ISFR) 2021-ISFR is
House contrary to any direction issued by his an assessment of India’s forest and tree cover
political party or anyone authorised to do so published every two years by the Forest Survey
without obtaining prior permission.
of India under the MoEFCC.
l As a pre-condition for his/her disqualification
his/her abstention from voting should not be OO India’s total forest and tree cover is now spread
condoned by his party or the authorised person across 80.9 million hectares, which is 22 per cent
within 15 days of such incident. of the geographical area of the country.
MOCK TEST PAPER - 01 25

OO Area wise, Madhya Pradesh has the largest forest of writs that can be issued by the courts. They are as
cover in the country followed by Arunachal follows:
Pradesh, Chhattisgarh, Odisha and Maharashtra. Habeas Corpus:
OO The top five states in terms of increase in forest
It’s a Latin phrase that literally means “to have the
cover are Andhra Pradesh (647 sq km), Telangana body of.” It is a court order requiring a person who
(632 sq km), Odisha (537 sq km), Karnataka (155 has detained another person to produce the latter’s
sq km) and Jharkhand (110 sq km). body before it. The court then considers the reason
79. Option (c) is correct. for the detention as well as its legality. Both public
authorities and private individuals can be served
Explanation: A satellite launched from the sites near
with a writ of habeas corpus.
the equator towards the east direction will get an

The writ, on the other hand, is not issued where:
initial boost equal to the velocity of the Earth surface.
OO The detention is lawful,
The initial boost helps in cutting down the cost of
OO The proceeding is for contempt of a legislature
rockets used to launch the satellites. Escape velocity
or a court,
has nothing to do with the direction of earth’s
OO The detention is by a competent court, and
rotation. Escape velocity is the velocity required for
OO The detention is outside the court’s jurisdiction.
a body to escape from the gravitational pull of the
Mandamus
earth. It is greater near the surface and decreases as

It literally translates to ‘we command.’
it moves away from the surface. Hence, Assertion (A)

It is a court-issued directive to a public official,
is correct, but Reason (R) is not correct.
requesting that he perform his official duties which
80. Option (d) is correct.
he failed to perform or refused to do so.
Explanation: 3D printing is a process of making It can also be used against any public figure,
three-dimensional solid objects from a digital file. corporation, a lower court, a tribunal, or a government
The creation of a 3D printed object is achieved using for the purpose of the same goal.
additive processes. In an additive process, an object is A writ of mandamus cannot be issued:
created by laying down successive layers of material OO against a private individual or group
until the object is created. Each of these layers can OO against the president of India, state governors,
be seen as a thinly sliced horizontal cross-section of high court justice acting in a judicial capacity.
the eventual object. Few applications of 3D printing Prohibition
include consumer products like eyewear, footwear, It literally means ‘to forbid.’
industrial products, dental products, prosthetics, It is issued by a higher court to a lower court or
automative industry, data processing technologies tribunal to prevent the latter from exceeding its
etc. jurisdiction or usurping the jurisdiction that it lacks.
In contrast to the mandamus, which directs activity,
81. Option (a) is correct.
the prohibition directs inactivity.
Explanation: Writs come under the original Only judicial and quasi-judicial authorities can be
jurisdiction of the Supreme Court as well as the High served with a prohibition writ.
Court. Certiorari
Article 32 of the Constitution (Right to Constitutional It literally means ‘to be certified’ or ‘to be informed.’
Remedies): It is a fundamental right which states It is issued by a higher court to a lower court or
that individuals have the right to approach the tribunal to either transfer a case pending with the
Supreme Court (SC) seeking enforcement of other latter or to overturn the latter’s order in a case.
fundamental rights recognised by the Constitution. Quo-Warranto

Article 226 of the Constitution empowers a high It literally means ‘by what authority or warrants’.
court to issue writs including habeas corpus,
It is issued by the court to investigate the legality
mandamus, certiorari, prohibition, and quo warranto of a person’s claim to a public office. As a result, it
for the enforcement of the fundamental rights of the prevents a person from illegally usurping public
citizens and for any other purpose. The Supreme office.
Court can issue the writs only for the enforcement of
The writ can only be issued in the case of a substantive
fundamental rights and not for other purposes. public office of permanent character established by

The Indian Constitution specifies five different types statute or by the Constitution.
26 UPSC CSE 15 MOCK TEST PAPERS GENERAL STUDIES PAPER I

82. Option (c) is correct. 85. Option (c) is correct.

Explanation: A black hole is an object in space that is Explanation: These are the main styles of Indian
so dense and has such strong gravity that no matter temple architecture. Two broad orders of temples
or light can escape its pull. Because no light can in the country are known as Nagara in the north
escape, it is black and invisible. Any light or matter and Dravida in the south. At times the Vesara style
that crosses that boundary is sucked into the black of temples is also found as an independent style
hole. It would need to travel faster than the speed of created through the selective mixing of the Nagara
light to escape, which is not possible. and Dravida orders.

83. Option (c) is correct. 86. Option (d) is correct.


Explanation: The Common stock allows its holders
Mantle to make a profit through rising share prices and
dividend payments. Holders of common stock also
have voting rights, but holders of preferred stock do
not get a vote on company matters.
87. Option (a) is correct.
Explanation: Diwan-i-bandagan: Tughlaq

Diwan-i-Mustakhraj: Alauddin Khalji

Diwan-i-Kohi: Tughlaq

Diwan-i-Arz: Iltutmish

Important departments during Medieval history

Name of the Central Function of the


Department Department
Figure: Coriolos Effect Diwan-i-Risalat Department of appeals
Explanation: Dynamo theory proposes a mechanism Diwan-i-arz Department of Military
by which a celestial body such as Earth or a star Diwan-i-Ishtiaq Department of pensions
generates a magnetic field and sustains it over
Diwan-i-Mustakhraj Department of arrears
astronomical time scales (millions of years).

Dynamo theory suggests that convection in the Diwan-i-kohi Department of agriculture
outer core, combined with the Coriolis effect (caused Diwan-i-insha Department of
due to the rotation of the earth), gives rise to self- correspondence
sustaining (geodynamo) Earth’s magnetic field. Diwan-i-Bandagan Department of slaves

The Earth’s magnetic field has reversed every few
Diwan-i-Qaza-i- Department of justice
hundred thousand years, but there is no apparent
Mamalik
periodicity to the occurrence. The reversal of polarity
is a very random event. Diwan-i-Khairat Department of charity

At the time when the earth was created, there was 88. Option (a) is correct.
no atmosphere. Initially, life might have started
anaerobic respiration, later on the process of Explanation:
photosynthesis and other processes slowly modified l Qutub Minar is built in 1193 by Qutab-ud-din
the early atmosphere of the Earth. Aibak.
l Krishna devaraya was an emperor of the
84. Option (a) is correct.
Vijayanagara Empire who reigned from 1509–
Explanation: Even though 71 percent of the Earth’s 1529 AD. He was the third ruler of the Tuluva
surface is covered by water. More than 99 percent of Dynasty and is widely considered to be the
it (Earth’s water) is unusable by humans and many greatest ruler of the empire. He was also known
other living things. Only about 0.3 percent of our for employing Turkish archers.
fresh water is found in the surface water of lakes, l The first Portuguese encounter with the
rivers are usable. subcontinent was on 20 May 1498 when Vasco

Over 68 percent of the fresh water available on the da Gama reached Calicut on the Malabar Coast.
Earth surface is found in ice caps and glaciers, and Sultan Firoz Shah Tughlaq ruled from 1309 AD to
just over 30 percent is found in ground water. 20 September 1388 AD.
MOCK TEST PAPER - 01 27

89. Option (b) is correct.


Current account consists of short-term transaction
which includes balance of trade, balance of invisible
Explanation: William Hawkins reached the court interest payment and remittance.
of Jahangir in 1608 AD to seek permission for trade.
FDI that is foreign direct investment, foreign portfolio
The British thus established their first (temporary) investment, foreign loans and non resident deposits
factory at Masulipatnam in 1611. are part of capital account.

But, the first permanent British factory was
93. Option (d) is correct.
established at Surat in 1613.
Explanation: Inflation benefits the creditor as
90. Option (b) is correct.
the net return given by creditors is less because
Explanation: inflation erodes the value of money. Similar holds
OO The Charter Acts of 1813 was an act that renewed true for bondholders. Recently, the government has
the charter issued to the British East India announced that it will issue an inflation index bond
Company and continued the Company’s rule where the inflation rate is adjusted to the principal
in India. However, the Company’s commercial amount, and thus, in case of deflation, the principal
monopoly was ended except for the tea trade amount will not be lost thus attracting more
and the trade with China. investment and also protecting the investors from
The Charter Act of 1853: inflation effect. Such types of bonds were also issued
OO Patronage system to civil service ended. in 1997 by the name of capital indexed bond.
OO The Court of directors strength was reduced from
94. Option (a) is correct.
24 to 18 out of which 6 people were nominated
by the British crown. Separated the governor- Explanation: Liquidity is a concept which helps us
general of Bengal from GGI. to understand how is easily an asset is converted into
OO The legislative wing came to be known as the cash. Currency in circulation is most liquid which is
Indian Legislative Council. followed by saving deposit. In the given options,
Government of India Act 1858: time deposit are least liquid asset as they have fixed
OO In August 1858 the British parliament passed an maturity period.
act that set an end to the rule of the company. 95. Option (a) is correct.
The control of the British government in India
Explanation: Standing facility is defined as a rate at
was transferred to the British crown.
which RBI gives loan to scheduled commercial banks
OO Changed designation from Governor general
in emergency, mostly overnight.
to viceroy. New secretary of state in British
parliament to supported India council. Lord
The security is provided under MSF (Marginal
canning first viceroy. Standing Facility) are those which are used under
SLR (Statutory Liquidity Ratio).
91. Option (c) is correct.
96. Option (a) is correct.
Explanation: The World Economic Outlook is
Explanation: Agreement on agriculture is one of
released by the International Monetary Fund (IMF).
the most important agreement related to the WTO
IMF is also responsible for releasing the Global
(World Trade Organisation) which aimed at in oven
Financial Stability Report.
trade barriers and promoting transparent market
Reports released by the Organisation of Economic
access and integration. The subsidies are clubbed
Development and Cooperation are:
into three groups.
1. The program for international student

Green box which include subsidies on which there is
assessment also known as Pisa.
no limit, and they are not considered trade distorting.
2. Global index of countries. Generally, income support schemes like PM Kisan
3. Government at a glance report. are considered in green box.
92. Option (b) is correct.
Amber box subsidy covers all those support which
Explanation: There are two main accounts in the are considered production distorting. They are to
BoP (balance of payment) are the current account be maintained within 5% for the developed country
and 10% for the developing country based on their
and the capital account.
production value.
Current Account: The current account records

Blue box subsidy is direct payment under production
exports and imports in goods, trade in services and
limiting program, they are also considered as trade
transfer payments.
distorting.
Capital Account: The capital account records all
97. Option (c) is correct.
international purchases and sales of assets such
as money, stocks, bonds, etc. It includes foreign Explanation: When RBI is following an expansionary
investments and loans monetary policy, it means it is aiming at increasing
28 UPSC CSE 15 MOCK TEST PAPERS GENERAL STUDIES PAPER I

the liquidity in the market. Increasing the CRR will and planned expenditure has also lost their relevance
compels the bank to reduce their lending, thus is not within the aspect of planning.
a part of expansionary monetary policy. Increasing 99. Option (a) is correct.
the SLR will also compels the bank to lend less.
Explanation: A decrease in the rate of interest of
Frequent use of open market operations can be used
lending will help in easy availability of money, thus it
for both expansionary monetary policy and tight
will increase the investment expenditure within the
monetary policy.
economy. Increase in non-tax revenue is not directly

Decreasing the repo rate will help scheduled
related to decrease in the rate of interest. Similarly,
commercial banks to get cheaper loans from the RBI,
the increase in FDI and increase in global export
thus will encourage the banks to give more and more
have nothing to do with the decreased interest rate
loans, which will increase the liquidity in the market.
within an economy.
98. Option (d) is correct.
100. Option (c) is correct.
Explanation: Non-planned expenditures which are
Explanation: The National income of an economy
not included in five-year plan.
in the simplest form is defined as total monetary

The biggest items of Non-Plan Expenditure are
value of final goods and services produced in a
interest payments and debt servicing, defence
geographical area within a time period. The most
expenditure, salaries and subsidies.
important indicators used for calculation of national

But, with the removal of the planning commission,
income are GDP, GNP, NDP, NNP etc.
the distinction between non-planned expenditure

❑❑

You might also like